Вы находитесь на странице: 1из 31

VISIONIAS

www.visionias.in
ANSWERS & EXPLANATION
GENERAL STUIDES (P) – 2685 (2019)

Q 1.B
 The Preamble in its present form reads:
 ―We, THE PEOPLE OF INDIA, having solemnly resolved to constitute India into a SOVEREIGN
SOCIALIST SECULAR DEMOCRATIC REPUBLIC and to secure to all its citizens:
 JUSTICE, Social, Economic and Political;
 LIBERTY of thought, expression, belief, faith and worship;
 EQUALITY of status and of opportunity; and to promote among them all;
 FRATERNITY assuring the dignity of the individual and the unity and integrity of the Nation;
 IN OUR CONSTITUENT ASSEMBLY this twenty-sixth day of November, 1949, do HEREBY ADOPT,
ENACT AND GIVE TO OURSELVES THIS CONSTITUTION‖.
 Ingredients of the Preamble: The Preamble reveals four ingredients or components:
o Source of authority of the Constitution: The Preamble states that the Constitution derives its
authority from the people of India.
o Nature of Indian State: It declares India to be of a sovereign, socialist, secular democratic and
republican polity.
o Objectives of the Constitution: It specifies justice, liberty, equality and fraternity as the objectives.
o Date of adoption of the Constitution: It stipulates November 26, 1949 as the date.

Q 2.C
 Jupiter‘s Great Red Spot — the solar system‘s most famous storm — is almost one-and-a-half Earths
wide and penetrates about 300 kilometres into the planet‘s atmosphere, according to data collected by
NASA‘s Juno spacecraft.

Q 3.A
 Statement 1 is not correct: Under Article 350A - It shall be the endeavour of every State and of every
local authority within the State to provide adequate facilities for instruction in the mother-tongue at the
primary stage of education to children belonging to linguistic minority groups; and the President may
issue such directions to any State as he considers necessary or proper for securing the provision of such
facilities.
 Under Article 350B - There shall be a Special Officer for linguistic minorities to be appointed by the
President. It shall be the duty of the Special Officer to investigate all matters relating to the safeguards
provided for linguistic minorities under this Constitution and report to the President upon those matters at
such intervals as the President may direct, and the President shall cause all such reports to be laid before
each House of Parliament, and sent to the Governments of the States concerned.
 Statement 2 is not correct: The Constitution specifies that Hindi written in Devanagari script is to be the
official language of the Union. But, the form of numerals to be used for the official purposes of the Union
has to be the international form of Indian numerals and not the Devanagari form of numerals. The
Constitution does not specify the official language of different states. It specifies that The legislature
of a state may adopt any one or more of the languages in use in the state or Hindi as the official
language of that state and it does not need the approval of Rajya sabha. Until that is done, English is
to continue as official language of that state. Under this provision, most of the states have adopted the
major regional language as their official language.
 Statement 3 is correct: The constitutional provisions dealing with the language of the courts and
legislation are as follows:
1 www.visionias.in ©Vision IAS
o Until Parliament provides otherwise, the following are to be in the English language only:
 All proceedings in the Supreme Court and in every high court.
 The authoritative texts of all bills, acts, ordinances, orders, rules, regulations and bye-laws
at the Central and state levels.

Q 4.C
 Asita Project (Asita is another name of Yamuna)
 It's a Yamuna River Front Development Project, aims to restore, revive and rejuvenate the river‘s
floodplains and make them accessible to the people of Delhi.
 Reviving Ecosystem by creating a wetlands, to store the flood waters and to improve the groundwater
recharge which will eventually result in flourishing of biodiversity in the floodplains.
 National Mission for Clean Ganga (NMCG) is monitoring the development of project.

Q 5.A
 Mass Movement: Mass movement or mass wasting is the term used for the movement of material down a
slope under the influence of gravity. Thus it excludes those in which material is carried directly by a
transporting medium such as running water, wind or ice. That means mass movement does not come
under erosion though there is a shift of materials from one place to another. Hence, statement 1 is
correct.
 The movement of mass may range from slow to rapid, affecting shallow to deep columns of materials and
include creep, flow, slide and fall. Weathering is not a pre-requisite for mass movement though it aids
mass movement. Thus, mass movement is more active over weathered slopes as compared to unweathered
slopes. Hence, statement 2 is not correct.

Q 6.B
 Gross fixed capital formation (GFCF) refers to the net increase in physical assets (investment minus
disposals) within the measurement period. It does not account for the consumption (depreciation) of
fixed capital, and also does not include land purchases. To be more precise Gross fixed capital formation
measures the net increase in fixed capital. It is a component of expenditure approach to calculating
GDP. Hence statement 1 is not correct.
 GDP as defined by the expenditure method consider the end use of incomes generated on expenditure
and investments. It is distinct from GDP estimated by economic activity, where the economy is divided
into segments like agriculture, industry and services and GDP is estimated by measuring the value added
at each stage of the production process.
 GDP as measured through the expenditure method is the sum of Private Final Consumption
Expenditure (PFCE), Government Final Consumption Expenditure (GFCE), Gross Fixed Capital
Formation (GFCF), Change in stocks or inventories, Valuables and the Net Exports (NX). Hence
statement 2 is correct.
 GFCF measures the addition of fixed assets through expenditure on construction, purchase of machinery
and equipment and software

Q 7.D
 A unique form of architecture developed in the hills of Kumaon, Garhwal, Himachal and Kashmir.
Kashmir‘s proximity to prominent Gandhara sites (such as Taxila, Peshawar and the northwest frontier)
lent the region a strong Gandhara influence by the fifth century CE. This began to mix with the Gupta and
post-Gupta traditions that were brought to it from Sarnath, Mathura and even centres in Gujarat and
Bengal. Hence statement 1 is correct.
 The Karkota period of Kashmir is the most significant in terms of architecture. One of the most important
temples is Pandrethan, built during the eighth and ninth centuries. In keeping with the tradition of a water
tank attached to the shrine, this temple is built on a plinth built in the middle of a tank. Although there are
evidences of both Hindu and Buddhist followings in Kashmir, this temple is a Hindu one, possibly
dedicated to Shiva. Few more temples in Kashmir, the Martand Sun Temple, the Awantipora temples, the
Sankara-Gauresvara temple, temple of Sugandhesa at Patan, the Pandrethan temples, the Shiva Bhutesa
and Siva Jeyshthesa temples at Vangath, the Parihasakesva, Muktakesva, Mahavarha and
Goverdhanadhara temples in Parihasapura, and the famous Mameswara Siva temple at Mamalaka are
some specimens of great archaeological value. Hence, statement 2 and 3 are correct

2 www.visionias.in ©Vision IAS


Q 8.C
 Under the ambit of Ayushman Bharat, a Pradhan Mantri Jan Arogya Yojana (PM-JAY) to reduce the
financial burden on poor and vulnerable groups arising out of catastrophic hospital episodes and ensure
their access to quality health services was conceived. PM-JAY seeks to accelerate India‘s progress
towards achievement of Universal Health Coverage (UHC) and Sustainable Development Goal - 3
(SDG3).
 Pradhan Mantri Jan Arogya Yojana (PM-JAY) will provide financial protection (Swasthya Suraksha) to
10.74 crore poor, deprived rural families and identified occupational categories of urban workers‘ families
as per the latest Socio-Economic Caste Census (SECC) data (approx. 50 crore beneficiaries). It will have
offer a benefit cover of Rs. 500,000 per family per year (on a family floater basis).
 Benefits of PM-JAY:
o Government provides health insurance cover of up to Rs. 5,00,000 per family per year.
o More than 10.74 crore poor and vulnerable families (approximately 50 crore beneficiaries)
covered across the country.(Hence, statement 3 is correct)
o All families listed in the SECC database as per defined criteria will be covered. No cap on family size
and age of members.(Hence, statement 1 is not correct)
o Priority to girl child, women and and senior citizens.
o Free treatment available at all public and empaneled private hospitals in times of need. (Hence,
statement 2 is correct)
o Covers secondary and tertiary care hospitalization.
o 1,350 medical packages covering surgery, medical and day care treatments, cost of medicines and
diagnostics.
o All pre-existing diseases covered. Hospitals cannot deny treatment.
o Cashless and paperless access to quality health care services.
o Hospitals will not be allowed to charge any additional money from beneficiaries for the treatment.
o Eligible beneificiares can avail services across India, offering benefit of national portability.

Q 9.C
 Article 123 of the Constitution empowers the President to promulgate ordinances during the recess of
Parliament. These ordinances have the same force and effect as an act of Parliament, but are in the nature
of temporary laws. The ordinance-making power is the most important legislative power of the President.
It has been vested in him to deal with unforeseen or urgent matters. But, the exercises of this power is
subject to the following four limitations:
 He can promulgate an ordinance only when both the Houses of Parliament are not in session or when
either of the two Houses of Parliament is not in session. An ordinance can also be issued when only one
House is in session because a law can be passed by both the Houses and not by one House alone. An
ordinance made when both the Houses are in session is void. Thus, the power of the President to legislate
by ordinance is not a parallel power of legislation.
 He can make an ordinance only when he is satisfied that the circumstances exist that render it necessary
for him to take immediate action.
 His ordinance-making power is coextensive as regards all matters except duration, with the law-making
powers of the Parliament. This has two implications:
 An ordinance can be issued only on those subjects on which the Parliament can make laws.
 An ordinance is subject to the same constitutional limitation as an act of Parliament. Hence, an ordinance
cannot abridge or take away any of the fundamental rights.
 Every ordinance issued by the President during the recess of parliament must be laid before both the
Houses of Parliament when it reassembles. If the ordinance is approved by both the Houses, it becomes an
act. If Parliament takes no action at all, the ordinance ceases to operate on the expiry of six weeks from
the reassembly of Parliament. The ordinance may also cease to operate even earlier than the prescribed six
weeks, if both the Houses of Parliament pass resolutions disapproving it. If the Houses of Parliament are
summoned to reassemble on different dates, the period of six weeks is calculated from the later of those
dates. This means that the maximum life of an ordinance can be six months and six weeks, in case of non-
approval by the Parliament (six months being the maximum gap between the two sessions of
Parliament). If an ordinance is allowed to lapse without being placed before Parliament, then the
acts done and completed under it, before it ceases to operate, remain fully valid and effective.

3 www.visionias.in ©Vision IAS


 The President can also withdraw an ordinance at any time. However, his power of ordinance-making is
not a discretionary power, and he can promulgate or withdraw an ordinance only on the advice of
the council of ministers headed by the prime minister.
 An ordinance like any other legislation, can be retrospective, that is, it may come into force from a back
date. It may modify or repeal any act of Parliament or another ordinance. It can alter or amend a tax law
also. However, it cannot be issued to amend the Constitution.

Q 10.D
 Statement 1 is correct: Globally Important Agricultural Heritage Systems (GIAHS) are outstanding
landscapes of aesthetic beauty that combine agricultural biodiversity, resilient ecosystems and a valuable
cultural heritage. Located in specific sites around the world, they sustainably provide multiple goods and
services, food and livelihood security for millions of small-scale farmers.
 Unfortunately, these agricultural systems are threatened by many factors including climate change and
increased competition for natural resources. They are also dealing with migration due to low economic
viability, which has resulted in traditional farming practices being abandoned and endemic species and
breeds being lost.
 These ancestral agricultural systems constitute the foundation for contemporary and future agricultural
innovations and technologies. Their cultural, ecological and agricultural diversity is still evident in many
parts of the world, maintained as unique systems of agriculture.
 Statement 2 is not correct: These sites as designated by the Food and Agriculture Organisation (FAO) of
the UN.
 Statement 31 is correct: India has 3 sites designated as GIAHS: Saffron Heritage of Kashmir, Koraput
Traditional Agriculture (Odisha) and Kuttanad Below Sea Level Farming System (Kerala).

Q 11.A
 At the 10th BRICS summit, the heads of states of the five countries adopted the Johannesburg
Declaration. It was held under the theme "BRICS in Africa: Collaboration for Inclusive Growth and
Shared Prosperity in the 4th Industrial Revolution‖.
 China introduced the ―BRICS Plus‖ format at the Xiamen summit last year by inviting a few
countries from different regions. South Africa emulated it, arranging the attendance of top-level
representation of five nations of its choice: Argentina, Jamaica, Turkey, Indonesia and Egypt. The precise
role of ―BRICS Plus‖ countries will take time to evolve.
 In the Xiamen declaration, the BRICS countries had called upon all nations to adopt a comprehensive
approach in combating terrorism including countering radicalisation and blocking terror financing
sources.

Q 12.A
 The Asian Highway network is a regional transport cooperation initiative aimed at enhancing the
efficiency and development of the road infrastructure in Asia, supporting the development of Euro-
Asia transport linkages and improving connectivity for landlocked countries.
 The Asian Highway network now comprises over 141,000 km of roads passing through 32 member
countries. The network extends from Tokyo in the east to Kapikule, Turkey in the west and from
Torpynovka , Russian Federation, in the north, to Denpasar, Indonesia in the south.
 The Asian Highway project was initiated in 1959 with the aim of promoting the development of an
international road transport system in the region. From 1960 to 1970, potential routes were identified and
analysed. However, the progress was slow until political and economic changes in the region spurred
renewed interest in the network in the late 1980s and early 1990s.
 The formalization of the network was initiated in 2002. The ESCAP secretariat worked with national
governments to develop the International Agreement on the Asian Highway Network, which was adopted
on 18 November 2003 and entered into force on 4 July 2005. The agreement includes a list of Asian
Highway routes and classification and design standards.
 India signed the Inter-Governmental Agreement on Asian Highway Network during the 60th
Annual Session of the United Nations Economic and Social Commission for Asia and Pacific
(UNESCAP), held in April, 2004.

4 www.visionias.in ©Vision IAS


 The Asian Highways (AH) 1 and AH 2 passes through the North-Eastern States of India following
the National Highway stretches from the Indo-Myanmar border at Moreh in the State of Manipur via
Imphal-Kohima (Nagaland)-Dimapur-Nagaon-Jorabat (Assam)-Shillong-Dawki- Tamabil upto Indo-
Bangaldesh border in the State of Meghalaya having a length of about 740 km.

Q 13.B
 The events and traditions of the middle Ganga plains where Magadha was prominently located are well
preserved in the early Buddhist and Jain literature. Some of the texts of the Buddhist tradition are
compiled as the Tripitikas and the Jatakas.
 Later Buddhist chronicles like the Mahavamsa and Dipavamsa compiled in Sri Lanka are significant
sources for the events related particularly to Ashoka's reign.
 Dipavamsa, (Pali: ―History of the Island‖), oldest extant historical record of Sri Lanka, compiled in the
4th century. It emphasises ecclesiastical (Buddhist) rather than political history.
o Unlike the Mahavaṃsa, the Dipavaṃsa is a crude, unpolished work—perhaps the first attempt of the
Sinhalese people to write in Pali.
o Because of its lack of organization and a heterogeneity of style, it is generally considered the product
of multiple authorship.
 The Mahavamsa—also written in Pali—is generally considered to be based on two main sources: the
Dipavaṃsa, and oral tradition handed down by Buddhist monks.
o Because of the inclusion in the Mahavaṃsa of much from these sources that is mythical or
supernatural, large portions of the text are of dubious historicity.
 Vimsatika is the short name of Vimsatika-vijnapti-matratasiddih Sastra. It was written by Vasubandhu
in Sanskrit.
o The Vimsatika contains twenty verses and a commentary by Vasubandhu, and is an exposition of
several important doctrines of the Yogacara School.

Q 14.B
 Lord Curzon gave priority to educational reform and on 27th January, 1902 appointed the Indian
Universities Commission to 'inquire into the condition and prospects of the universities established in
British India and to consider and report upon proposals on improving their constitution and working.‘
 The Commission submitted its report in 1902 itself. Dr. Gurudas Banerjee, the only Hindu member of
the Commission, expressed his strong disapproval of the Report to which was appended his 'Note of
Dissent‘.
 The Commission's recommendations on University education did not aim at any fundamental
reconstruction of the system, but merely at a rehabilitation and the strengthening of the existing system
and it was felt that the Government wanted to crush the college education through such means.
 When its report along with the 'Dissent Note' by Sir Gurudas Banerjee was published, it drew hostile
reactions from the leaders of education. There started a regular campaign of criticism of Report and
recommendations in the Indian press.
 Satish Chandra Mukherjee began a systematic attack of the Commission‘s recommendations in the
column of the leading nationalist daily Amrita Bazar Patrika and provided ‗a Scheme of Reform‘ of the
system in the pages of his magazine Dawn.
 He did not just stop at mere theoretical criticism but took the initiative of establishing in July 1902 a
special educational society and named it the Dawn Society, with the intent of remedying the defects and
deficiencies of the existing system of University education.
 The Dawn Society would be among other things ‗an institution for the cultivation of patriotic and
nationalistic impulses.‘
 Writing in the magazine in 1904, Satish Chandra Mukherjee dwelt on the need for inculcating a spirit of
unity which shall grow from the awareness of the richness and diversity of the nation and an empathy
towards fellow Indians; this he felt was what ought to be imparted by education, not the present existing
system, but a future one which is what the newly founded Dawn Society would ultimately aim at creating.
 But the Government of Lord Curzon paid no heed to the criticism of the press and the Universities‘ Bill
was finally passed on March 21 1904, on the basis of the majority Report of the Commission of 1902.
 It emphasized the need for maintaining a limited number of State institutions in each branch of education,
and that the Government should retain a general control over all public institutions through efficient
inspection.
5 www.visionias.in ©Vision IAS
Q 15.A
 Plantation agriculture was introduced by the Europeans in colonies situated in the tropics. Some of the
important plantation crops are tea, coffee, cocoa, rubber, cotton, oil palm, sugarcane, bananas and
pineapples.
 The characteristic features of this type of farming are large estates or plantations, large capital
investment, managerial and technical support, scientific methods of cultivation, single crop
specialisation, cheap labour, and a good system of transportation which links the estates to the factories
and markets for the export of the products.
 Overreliance on monsoons is not a characteristic of the plantation agriculture.

Q 16.C
 The Deputy Chairman or a member of the panel of Vice-Chairmen when presiding has the same powers
as the Chairman whenpresiding over the sitting of the House. It has been consistently held that no appeal
lies to the Chairman against a ruling given by the Deputy Chairman or any other member presiding over a
sitting of the House in the absenceof the Chairman. The ruling given from the Chair settles the matter
before the House and cannot be reopened. Hence statement 1 is correct.
 However, whenever a point raised in the House needs some consideration or involves application of
precedents or study, it is open to the Deputy Chairman or the Vice-Chairman to reserve the matter for the
consideration and decision of the Chairman. Hence statement 2 is correct.

Q 17.C
 The Kathmandu-based International Centre for Integrated Mountain Development‘s (ICIMOD) ―Hindu
Kush Himalaya Assessment‖ reveals that more than one-third of the glaciers in the region could retreat by
2100, even if the global temperature rise is capped at 1.5ºC.
 Since 1970s, nearly 15% of the glaciers in the HKH has disappeared. Eastern Himalaya glaciers have
tended to shrink faster than glaciers in the central or western Himalaya.
 In contrast to the Himalayan glaciers, on average, glacier areas in the Karakoram have not changed
significantly. Given the context of glacier retreat throughout the rest of the extended HKH region,
this behaviour has been designated the ‗Karakoram anomaly‘.

Q 18.C
 National Central Cooling Company PJSC (DFM: Tabreed), a leading UAE-based international cooling
provider, has entered into a 30 year concession to offer District Cooling solutions to the greenfield city of
Amaravati. The contracted cooling capacity for 20,000 refrigeration tons (RTs) entails build, own, operate
and transfer India‘s first district cooling system in Amaravati, the new capital of Andhra Pradesh.
 District cooling systems produce chilled water at a central plant and then pipe that energy out to buildings
for air conditioning. As a result, these buildings do not require their own chillers or air
conditioners. Hence statement 1 is correct.
 Compared to other cooling systems, district cooling uses only 50 per cent of the primary energy
consumption for cooling urban buildings, thereby reducing CO2 emissions. It also helps in improving the
air quality and reducing general noise levels when compared to other traditional air conditioning
systems. Hence statement 2 is correct.

Q 19.B
 Pair 1 is not correctly matched: Mohorovicic Discontinuity: The transition zone between the crust and
mantle is called as mohorovicic discontinuity. The Moho lies at the depth of 35km beneath the continents
and 8km beneath the oceanic crust. The Moho separates both the continental crust and the oceanic crust
from underlying mantle.
 Pair 3 is not correctly matched: Conorod Discontinuity: The transition zone between the upper and
lower part of the lithosphere, is called as Conorod discontinuity. It is detected seismically at about 10–12
km depth.The upper crust in the continental region consist of felsic rock such as granite and the lower one
consist of more magnesium rich mafic rocks such as basalt. In passing through the Conorod discontinuity
the velocity of longitudinal seismic waves increases abruptly from approximately 6to6.5km/sec.
 Pair 2 is correctly matched: Gutenberg Discontinuity: The mantle –core transition zone is called
Gutenberg discontinuity. This discontinuity lies at the depth of 2900km beneath the earth surface. In this
zone the velocity of seismic waves changes suddenly. The velocity of P wave decreases and S wave
completely disappear at this depth. S wave shear material and cannot transmit through liquid. So, it is
6 www.visionias.in ©Vision IAS
believed that the part above the discontinuity is solid and part beneath the discontinuity is in liquid or
molten form.

Q 20.C
 Lokpal and Lokayuktas Act (2013) seeks to establish the institution of the Lokpal at the Centre and the
Lokayukta at the level of the State and thus seeks to provide a uniform vigilance and anti-corruption road
map for the nation both at the Centre and at the States. The jurisdiction of Lokpal includes the Prime
Minister, Ministers, Members of Parliament and Groups A, B, C and D officers and officials of the
Central Government. Hence option d is correct.
 Provisions related to Removal and suspension of Chairperson and Members of Lokpal:
 The Lokpal shall not inquire into any complaint made against the Chairperson or any Member.
Hence option a is correct.
 The Chairperson or any Member shall be removed from his office by order of the President on grounds of
misbehaviour after the Supreme Court, on a reference being made to it by the President on a petition
signed by at least one hundred Members of Parliament has, on an inquiry held in accordance with the
procedure prescribed in that behalf, reported that the Chairperson or such Member, as the case may be,
ought to be removed on such ground. If the Chairperson or any Member is, or becomes, in any way
concerned or interested in any contract or agreement made by or on behalf of the Government of India or
the Government of a State or participates in any way in the profit thereof or in any benefit or emolument
arising there from otherwise than as a member and in common with the other members of an incorporated
company, he/she shall be deemed to be guilty of misbehaviour. Hence option b is correct and option c is
not correct.
 Not withstanding the above provisions, the President may, by order, remove from the office, the
Chairperson or any Member if the Chairperson or such Member, as the case may be,
o is adjudged an insolvent; or
o engages, during his term of office, in any paid employment outside the duties of his office; or
o is, in the opinion of the President, unfit to continue in office by reason of infirmity of mind or body.

Q 21.C
 The practice of inscribing imperial proclamations on stone is a marked feature of Ashoka‘s reign.
 Ashoka‘s inscriptions are divided into various categories.The two main categories are of the major rock
edicts and the pillar edicts. The rock and pillar edicts are sets of inscriptions that occur, with minor
variations, in different places.
 There are also several minor rock edicts, minor pillar edicts, and cave inscriptions. The minor rock edicts
are considered among the earliest inscriptions, the major rock edicts later than them, and the pillar edicts
still later.
 What makes Ashoka‘s edicts unique is that unlike royal inscriptions of later times, which follow a
conventional pattern and phraseology, Ashoka‘s inscriptions reveal the voice and ideas of the king.
 The set of 14 major rock edicts (or portions thereof) occur at:
o Kandahar (in Kandahar district, south Afghanistan) (only portions of rock edicts 12 and 13)
o Shahbazgarhi (Peshawar district, North-West Frontier Province, Pakistan)
o Mansehra (Hazara district, NWFP, Pakistan)

7 www.visionias.in ©Vision IAS


o Kalsi (Dehradun district, Uttarakhand)
o Girnar (Junagadh district, Gujarat)
o Bombay–Sopara (originally at Sopara in Thane district, Maharashtra; now in the Chhatrapati Shivaji
Maharaj Vastu Sangrahalaya, Mumbai; only fragments of rock edicts 8 and 9)
o Dhauli (Puri district, Orissa; separate rock edicts 1 and 2 replace major rock edicts 11–13)
o Jaugada (Ganjam district, Orissa; separate rock edicts 1 and 2 replace major rock edicts 11–13)
o Erragudi (Kurnool district, Andhra Pradesh)
o Sannati (Gulbarga district, Karnataka; portions of rock edicts 12 and 14 and separate rock edicts 1
and 2 were found on a granite slab in a medieval goddess temple.
 The set of six (and in one case seven) pillar edicts, or portions thereof, occur at:
o Kandahar (Kandahar district, south Afghanistan) (only portions of pillar edict 7)
o Delhi. The Delhi–Topra pillar originally stood in Topra (Ambala district, Haryana). This pillar has
seven edicts.
o Delhi. The Delhi–Meerut pillar originally stood in Meerut (Meerut district, UP).
o Allahabad. The Allahabad–Kosam pillar was probably originally located in Kosam, i.e., Kaushambi
(Allahabad district, UP).
o Lauriya–Araraj (Champaran district, Bihar)
o Lauriya–Nandangarh (Champaran district, Bihar)
o Rampurva (Champaran district, Bihar)

Q 22.B
 Bond rating agencies are companies that assess the creditworthiness of both debt securities and their
issuers. The ratings are published by credit rating agencies and used by investment professionals to assess
the likelihood that the debt will be repaid. Rating shopping occurs when an issuer chooses the rating
agency that will assign the highest rating or that has the most lax criteria for achieving a desired
rating.
 Rating shopping has a strong effect when one rating agency's criteria is much more lax than
its competitors' criteria. Unless investors demand multiple ratings on deals, issuers will tend to use
only ratings from the agency with the most lenient standards.

Q 23.A
 The Island of Nalabana which is 8 km in circumference occupies a unique place in the vast expanse of
Chilika Lake, Odisha.
 The island possess Nalabana Bird Sanctuary which is one of the major attractions of Chilika. The island
is a protection site that preserves contrasting species of the migratory birds. The sanctuary is home to
thousands of migratory and regional bird species.
 Waterfowl such as Flamingos are the highlighting avian species arriving at this island. Green Sandpiper,
Tringa Ochropus, Halaban, Spoonbilled Sandpiper and Limnodromus Semipaimatus are some of the
noteworthy species of the sanctuary.

Q 24.C
 Oumuamua, the first interstellar object known to enter our solar system, accelerated faster away
from the sun than expected, hence the notion that some kind of artificial sail that runs on sunlight —
known as a light sail — may have helped push it through space.
 Oumuamua, Hawaiian for ―messenger‖ or ―scout,‖ was first viewed by telescopes in October 2017. The
alien rock is about 1,300 feet long (400 metre) long, and only about 130 feet wide.

Q 25.C
 Current account is a part of the overall balance of payment and reflects a country‘s international trade. It
primarily measures net trade in goods and services along with all other earnings or payments that are
required to be completed in a defined time period. Latest data shows India has a $48 billion current
account deficit. India‘s exports, which are much lower than imports, is the main cause of current account
deficit. Another major component of India‘s deficit is foreign investment income, where profits are
repatriated to a company‘s origin country. India is in surplus in trade in services and a net gainer of
remittances. These two surplus components, however, are not large enough to offset the trade deficit.
 The detailed analysis of service component of current account deficit shows that the largest component of
India‘s services surplus comes from IT industries. Similarly, India is a net exporter of travel meaning
8 www.visionias.in ©Vision IAS
foreigners visiting India spend more money than Indians visiting foreign countries. India has to send
abroad a significant amount of money for use of intellectual property. India is a net importer of
recreational services that include services in film, music industry and so on.

Q 26.D
 Defence: Currently FDI up to 49 per cent is permitted in the sector through automatic route and beyond
that up to 100 per cent via government nod is permitted.
 Pharma Sector: The government in 2016 allowed up to 74 per cent foreign direct investment in the
existing pharmaceutical companies through automatic route, with an aim to promote the sector. 100 per
cent FDI is allowed under automatic route in greenfield pharma and up to 100 per cent under government
approval in brownfield pharma.
 Banks: Currently, 20 per cent foreign investment is permitted in PSU banks under government approval
route.However the same is 74 per cent for private sector banks provided there is no change of control and
management of the investee company.

Q 27.C
 The kingdom of Mysore lay south of Hyderabad. In the 18th century, the rulers of Mysore, from the
Wodeyars to Tipu Sultan, faced the expansionist threat of the Marathas on the one hand and that of
Hyderabad and Carnatic on the other, while the English also exploited the situation to their advantage.
 The local chiefs, mainly poligars, were descendants of the hunter-gatherers of the forests who had
acquired military skills and local political leadership in the military service of the Vijaynagar empire.
 Haidar Ali auctioned off large territories to ambitious warriors, who as tax farmers, pressed revenue
demands upon these local chiefs. Haidar Ali refused these chiefs any claim to independence and if they
resisted they were driven off their lands. By limiting the scope of these chiefs' activities, Haidar further
eroded their local base.
 Tipu Sultan, his son, went further in the subjugation of the poligars. After expelling them he rented out
their lands to either private individuals or government officials. Further, by insisting to pay a regular
salary to his troops rather than pay them with spoils of war, Tipu was able to ensure that no vested
interest could emerge in the army in a tie up with the local chiefs.
 The land revenue policy under Tipu envisaged independent individual initiative to develop facilities for
agriculture. Rent free land was gifted to individuals for the construction of irrigation and other
infrastructure. Thus a class of people who could support agricultural development independently was
sought to be created.
 Tipu realised the importance of trade. He appointed asufs to train officials to run trade centres established
by him for keeping trade in control. Trade capital was to be provided for these trade centres from the
revenue collected by the state officials. Private traders were allowed to participate here in sale of
commodities thought to be beneficial to the state.

Q 28.B
 The Constitutional Proposals of the Sapru Committee, commonly referred to as the Sapru Committee
Report, was first published in 1945. It was prepared by a committee appointed by the Non-Party
Conference in November 1944.
o Tej Bahadur Sapru, a well-renowned lawyer, convened the first meeting of the Non-Party Conference
in 1941. This group consisted of individuals who represented a variety of interests except those
of the Indian National Congress, Muslim League and the Communist Party.
o The Committee was briefed ‗...to examine the whole communal and minorities question from a
constitutional and political point of view, put itself in touch with the different parties and their leaders,
including the minorities interested in the question, and present a solution...‘
o The Sapru Committee consisted of thirty members who had distinguished themselves in public
affairs and did not operate under the mandate of any political party.
 The Report contained detailed expositions on various aspects of India‘s constitutional future.
o The Report rejected the Muslim League‘s demand for Pakistan: it was not convinced that a separate
state would be advantageous to any community and felt that the division of India would ‗endanger the
peace and progress of the whole country‘.
o It called for the setting up of a constitution-making body that would frame a new constitution
for India; representation of Muslim and Hindus in this body would be equal.

9 www.visionias.in ©Vision IAS


o It rejected separate electorates for Muslims for the Union Legislature and instead proposed joint
electorates with reservation of seats.
o It introduced a ‗Minorities Commission‘ that would assess the welfare of minorities and had powers
to recommend measures to the government.
 The Report had a section on fundamental rights (similar to constitutional antecedent documents
that preceded it like the Nehru Report, 1928) and contained provisions that included: freedom of
speech, freedom of press, religious freedom and equality. It called for the future constitution body
to precisely formulate these rights.
o In the explanatory sections of the study, the Committee engaged with the issue of dividing rights into
justiciable and non-justiciable though it did not propose anything in its recommendation; it alerted
future drafters of the question to pay attention to this question. This was arguably the first time a
constitutional document brought up the question of justiciable and non-justiciable rights.

Q 29.B
 Mumbai has recently became the first Indian city on the World Cities Culture Forum
(WCCF).Mumbai, like members of the Forum, will be able to share its culture as part of a comparative
research to understand its role and impact. The municipal corporation will be able to maintain a
relationship with the other member cities and Mumbai will be represented on the Forum at all events.
 The World Cities Culture Forum: It is a network of local governments and cultural sector leaders from
39 world cities.The World Cities Culture Forum was established in London in 2012 with eight cities. The
WCCF enables the policy makers of member cities to share research and intelligence, while exploring the
vital role of culture in prosperity. Forum members collaborate via a program of events including themed
symposia, regional summits and workshops.

Q 30.C
 Article 3 authorises the Parliament to:
o form a new state by separation of territory from any state or by uniting two or more states or parts of
states or by uniting any territory to a part of any state,
o increase the area of any state,
o diminish the area of any state,
o alter the boundaries of any state, and
o alter the name of any state.
 However, Article 3 lays down two conditions in this regard: one, a bill contemplating the above
changes can be introduced in the Parliament only with the prior recommendation of the President;
and two, before recommending the bill, the President has to refer the same to the state legistature
concerned for expressing its views within a specified period.
 The President (or Parliament) is not bound by the views of the state legislature and may either
accept or reject them, even if the views are received in time. Further, it is not necessary to make a fresh
reference to the state legislature every time an amendment to the bill is moved and accepted in Parliament.
In case of a union territory, no reference need be made to the concerned legislature to ascertain its views
and the Parliament can itself take any action as it deems fit.
 Moreover, the Constitution (Article 4) itself declares that laws made for admission or establishment of
new states (under Article 2) and formation of new states and alteration of areas, boundaries or names of
existing states (under Articles 3) are not to be considered as amendments of the Constitution under Article
368. This means that such laws can be passed by a simple majority and by the ordinary legislative
process.

Q 31.B
 The Reserve Bank of India‘s attempt to inject rupee liquidity through foreign exchange swap auction has
received good response. The RBI has different tools through which it injects liquidity into financial
markets. The ‗swap auction‘ which was conducted recently is one such tool. Hence statement 2 is
correct. This is being done to increase the supply of rupees in the market. Technically, this activity is
termed as a USD/INR buy/sell swap auction.
 Through this auction, the RBI will buy US dollars from banks. Hence statement 1 is not correct. In
turn the RBI will pay rupees to the participating banks at the current spot rate. At an average spot rate of
70 per dollar, the RBI will able to infuse about ₹35,000 crore into the system through this auction
process.
10 www.visionias.in ©Vision IAS
Simultaneously, the banks will agree to buy-back the same amount of dollars from the RBI after three
years — the tenor of this auction. The participating banks have to bid in the auction by quoting a forward
premium in terms of paisa that they will pay to buy back the dollars. For example, if the spot exchange
rate is 70 to a dollar, say Bank A quotes a premium of 150 paisa and bids for $25 million. So, the bank
will get ₹175 crore ($25 million multiplied by the exchange rate of 70). After three years, the bank has to
pay back approximately ₹179 crore ($25 million multiplied by the exchange rate of 71.5) to the RBI to
buy back $25 million.
 The dollar amount mobilised through the auction would reflect in the RBI‘s foreign exchange reserves for
the tenor of the swap as also in the RBI‘s forward liabilities.

Q 32.D
 There are two kinds of banking licences that are granted by the Reserve Bank of India – Universal Bank
Licence and Differentiated Bank Licence. Differentiated Banks (niche banks) are banks that serve the
needs of a certain demographic segment of the population. Small Finance Banks and Payment Banks are
examples of differentiated banks in India.
 Differentiated licensing refers to the system of different licenses in contrast to the existing universal bank
(SBI, ICICI etc). The universal banks including the PSBs and private sector banks can provide all banking
services and products. On the other hand, under differentiated banking license, the Small Finance Banks
and Payments Banks can provided only selected products (and in prescribed geographies). Hence
statement 1 is not correct.
 The scheduled commercial banks are those banks which are included in the second schedule of RBI Act
1934 and which carry out the normal business of banking such as accepting deposits, giving out loans and
other banking services. They include Universal banks such as Public Sector Banks, Private Banks,
Foreign Banks and Regional Rural Banks. Whie Regional Rural Banks are a sort of differentiated bank (as
they have their own niche and operate in specific locations) others have been granted Universal Banking
licences. Hence statement 2 is not correct.

Q 33.C
 When the ruler of Sambalpur, Narayan Singh died on 10th September, 1849 leaving no son to succeed
him, the Governor General Lord Dalhousie annexed Sambalpur by applying the Doctrine of Lapse.
 Earlier, since 1827 AD both Balaram Singh and his nephew Surendra Sai had been claiming the throne
of Sambalpur as the legal heir apparent. But the British authority repeatedly ignored the claim of
Surendra Sai.
 The British Government implicated Surendra Sai in a case of murder committed by his followers,
and arrested him and his uncle Balaram Singh and brother Udyanta Sai. They were sent to
the Hazaribag Jail as prisoners for life in 1840 AD.
 After annexation under the Doctrine of lapse , the British took some measures which caused
dissatisfaction among the Zamindars as well as the common people of Sambalpur.
 When the revolt of 1857 took place, then the accumulated discontentment of the tribal people of
Sambalpur burst into open rebellion under the leadership of Surendra Sai. Surendra Sai spent as many
as 17 years till the mutineers broke open the jail in 1857.
 On the night of 31st October, 1857, he appealed to the people to join him in liberating Sambalpur from the
British Paramountcy. Many tribals and tribal Zamindars joined hands with him.
 Surendra Sai organized the rebels into several groups in different places. Sambalpur's connection with
Hazaribagh, Ranchi, Cuttack and Nagpur were for sometime cut off by the rebels. For sometime the
situation in Sambalpur became uncontrollable for the British authorities.
 In April 1861, Major Impey was appointed as the Deputy Commissioner of Sambalpur. He firmly
believed that only conciliatory measures would induce the rebels including the leader Surendra Sai to
surrender. Surendra Sai started negotiation with the British authorities, and surrendered to Major
Impey on 16th May, 1862. He was granted a pension of Rs.1200/- per annum.
 However, Major Impey died at Sambalpur in December 1863 and in his place Captain Cumberledge
joined as Deputy Commissioner. In the night of 23rd January, 1864 Surendra Sai, his son and close
followers were arrested and sentenced to deportation for life. Although a higher court overturned this
conviction, Surendra Sai and six other prisoners were detained at Nagpur under Regulation III of 1818.
 The British officers feared that the presence of Surendra Sai and his followers would unsettle the mind of
the people and disturb the tranquility in the district of Sambalpur. They were kept in the Nagpur Jail till

11 www.visionias.in ©Vision IAS


April, 1866 and thereafter sent to the Fort of Asirgarh, where Surendra Sai breathed his last on 28th
February, 1884.
 Note: The Union Cabinet has recently approved renaming of Jharsuguda Airport, Odisha as ―Veer
Surendra Sai Airport, Jharsuguda."

Q 34.D
 Mariculture is a specialized branch of aquaculture involving the cultivation of marine organisms
for food and other products in the open ocean, an enclosed section of the ocean, or in tanks, ponds
or raceways which are filled with seawater. An example of the latter is the farming of marine fish,
including finfish and shellfish like prawns, or oysters and seaweed in saltwater ponds. Non-food
products produced by mariculture include: fish meal, nutrient agar, jewellery (e.g. cultured pearls), and
cosmetics.
 Recently, a draft national policy on mariculture mooted the idea of mariculture zones by demarcating
special areas in the sea for activities such as cage farming, bivalve farming, pen culture, seaweed culture,
hatcheries and nurseries based on scientific criteria.
 The draft policy was formulated by an expert committee formed by the National Fisheries Development
Board (NFDB) with A Gopalakrishnan, Director of Central Marine Fisheries Research Institute (CMFRI),
as Chairman. The policy aims to enhance mariculture production in the country and increase income and
employment opportunities in a sustainable way, in addition to promoting entrepreneurship by facilitating
technical and financial inputs.

Q 35.C
 The Pallavas reached their zenith during the reign of Mahendravarman I (c. 600–630), a contemporary of
Harsha and Pulakeshin II. The struggle for Vengi between the Pallavas and the Chalukyas became the
immediate pretext for a long, drawn-out war, which began with the defeat of the Pallavas.
o Mahendravarman‘s successor, Narasimhavarman I (reigned c. 630–668), also called Mahamall
or Mamalla, avenged the Pallava defeat by capturing Vatapi. He assumed the title of
Vatapikonda.
 The Chola ruler Parantaka succeeded his father Aditya I on the throne in 907 and ruled for forty-
eight years. Soon after his accession, as early as 910, he invaded the Pandyan country and assumed
the title Maduraikonda (‗ Capturer of Madura ‘).
o The Pandyan ruler at the time, Maravarman Rajasimha II (900-20) appealed for help to Kassapa V,
ruler of Ceylon, who sent an army to his aid. In due course, however, Parantaka defeated the
combined armies at the battle of Vellur.
o Rajasimha had to flee to Ceylon and the Chola conquest of the Pandya country was completed soon
after.
o Some years later, in the reign of Udaya IV of Ceylon (940-53), Parantaka made an unsuccessful
attempt to capture the insignia of the Pandyan king. His failure was remembered and made up for
several years later by his powerful descendant Rajendra I.

Q 36.B
 In 1850, the British passed the Caste Disabilities Removal Act. It is generally known as the Lex Loci
Act. This Caste Disabilities Removal Act extended the principle of Section 9 of Regulation VII of 1832 of
the Bengal Code to embrace the whole of India : protecting the right of a convert to inherit ancestral
property.
 By this enactment, it was declared that renunciation of or exclusion from religion or caste will not affect
rights of property or inheritance.
 Hence it set aside the provisions of Hindu law which had heavy penalties for renunciation of religion or
exclusion from caste.
 The passing of this Act was a step towards British interference in the social system of the Hindus.
 This was the first such enactment made on their own initiative. The former measures against Sati were
passed mainly at the instance and on the representation of the Indians themselves.
 In India, meetings were held against the Act and a petition was presented to the House of Lords. But the
Government paid no heed to this and the Lex Loci Act was enforced.
12 www.visionias.in ©Vision IAS
Q 37.D
 The Directive Principles of State Policy (DPSPs) are enumerated in Part IV of the Constitution from
Articles 36 to 51.
 To protect monuments, places and objects of artistic or historic interest which are declared to be of
national importance is a DPSP provided under Article 49 of the Constitution.
 Apart from the Directives included in Part IV, there are some other Directives contained in other Parts of
the Constitution. They are:
o Claims of SCs and STs to Services: The claims of the members of the Scheduled Castes and the
Scheduled Tribes shall be taken into consideration, consistently with the maintenance of efficiency of
administration, in the making of appointments to services and posts in connection with the affairs of
the Union or a State (Article 335 in Part XVI).
o Instruction in mother tongue: It shall be the endeavour of every state and every local authority
within the state to provide adequate facilities for instruction in the mother-tongue at the primary stage
of education to children belonging to linguistic minority groups (Article 350-A in Part XVII).
o Development of the Hindi Language: It shall be the duty of the Union to promote the spread of the
Hindi language and to develop it so that it may serve as a medium of expression for all the elements
of the composite culture of India (Article 351 in Part XVII).

Q 38.A
 Statement 1 is correct: The Alliance for an Energy Efficient Economy (AEEE) under the leadership of
the Bureau of Energy Efficiency (BEE) and NITI Aayog has recently released the first Nationwide ‗State
Energy Efficiency Preparedness Index‘. It examines states‘ policies and regulations, financing
mechanisms, institutional capacity, adoption of energy efficiency and energy savings. As part of its
commitment to create awareness about energy efficiency as a resource and also to develop an action plan
for energy conservation initiatives, Bureau of Energy Efficiency (BEE) and Alliance for an Energy
Efficient Economy (AEEE), released the ‗State Energy Efficiency Preparedness Index‘, which assesses
state policies and programmes aimed at improving energy efficiency across various sectors. The
nationwide Index is a joint effort of the NITI Aayog and BEE.
 The State Energy Efficiency Preparedness Index has 63 indicators across Building, Industry,
Municipality, Transport, Agriculture and DISCOM with 4 cross-cutting indicators. States are categorised
based on their efforts and achievements towards energy efficiency implementation, as ‗Front Runner‘,
‗Achiever‘, ‗Contender‘ and ‗Aspirant‘.
 Statement 2 is not correct: The ‗Front Runner‘ states in the inaugural edition of the Index are: Andhra
Pradesh, Kerala, Maharashtra, Punjab, and Rajasthan based on available data.
 The Index examines states‘ policies and regulations, financing mechanisms, institutional capacity,
adoption of energy efficiency measures and energy savings achieved. The required data was collected
from the concerned state departments such as DISCOMs, Urban Development Departments, etc., with the
help of State Designated Agencies (SDAs) nominated by the BEE. Apart from the data furnished by
SDAs, the AEEE also collected information from various central government sources such as: Central
Electricity Authority (CEA) General Review; Ministry of Road Transport and Highways (MoRTH)
annual report on the performance of State Road Transport Undertakings (SRTU); BEE programme
implementation reports; Petroleum Conservation Research Association (PCRA), Ministry of New and
Renewable Energy (MNRE); and Energy Efficiency Services Limited (EESL).

Q 39.C
 The Pradhan Mantri Swasthya Suraksha Yojana (PMSSY) was announced in 2003 with objectives
of correcting regional imbalances in the availability of affordable/ reliable tertiary healthcare
services and also to augment facilities for quality medical education in the country.
 PMSSY has two components:
 Setting up of AIIMS like Institutions
 Upgradation of Government Medical College Institutions
 A new AIIMS, in Haryana, was recently announced in the Interim Budget Speech 2019-20, under Phase-
VIII of the program.

Q 40.A
 All these are coastal landforms of submergence type.
 Fiord coasts: Fiords are submerged U-shaped glacial troughs. They mark the paths of glaciers that
plunged down from the highlands. They have steep walls, often rising straight from the sea, with tributary

13 www.visionias.in ©Vision IAS


branches joining the main inlet at right angles. Due to the greater intensity of ice erosion fiords are deep
for great distances inland but there is a shallow section at the seaward end formed by a ridge of rock.
 Dalmatian coast: This is the longitudinal coast where mountains run parallel or concordant to the coast.
The name is taken from the coast of Dalmatia, Yugoslavia, along the Adriatic Sea. The Dalmatian type of
coast is also typical of the Pacific coast.
 Ria coast: In upland coastal regions where the mountains run at right angles to the sea, that is transverse
or discordant to the coast, a rise in the sea level submerges or drowns the lower parts of the valleys to
form long, narrow branching inlets separated by narrow headlands. Such coasts are known as Ria coasts.
They differ from fiords in two important respects, i.e. they are not glaciated, and their depth increases
seawards. A ria coast is typical of the Atlantic type of coast.

Q 41.C
 Statement 1 is correct: Methanogens are the only known microorganisms that produce methane as a
metabolic byproduct in hypoxic conditions. (Hypoxia is a condition in which the body or a region of the
body is deprived of adequate oxygen supply at the tissue level.)
o Process of methane production is known as Methanogenesis
o They are common in wetlands, where they are responsible for marsh gas, and in the digestive tracts of
animals such as ruminants and humans, where they are responsible for the methane content of
belching in ruminants and flatulence in humans.
o Among livestock, methane production is greatest in ruminants, as methanogens are able to produce
methane freely through the normal process of feed digestion.
 Statement 2 is correct:
o Methanogens are strict anaerobes that do not use oxygen to respire; in fact, oxygen inhibits the growth
of methanogens.

Q 42.A
 Global Human Footprint Index represents the relative human influence in each terrestrial biome expressed
as a percentage. The purpose is to provide an updated map of anthropogenic impacts on the environment
in geographic projection which can be used in wildlife conservation planning, natural resource
management, and research on human-environment interactions. The index is prepared by the Wildlife
Conservation Society (WCS) and the Columbia University Center for International Earth Science
Information Network (CIESIN).
 WCS envisions a world where wildlife thrives in healthy lands and seas, valued by societies that embrace
and benefit from the diversity and integrity of life on earth. WCS saves wildlife and wild places
worldwide through science, conservation action, education, and inspiring people to value nature. WCS's
goal is to conserve the world's largest wild places in 16 priority regions, home to more than 50% of the
world's biodiversity.

Q 43.B

From the table given above, it can be concluded that statement 2 is correct.
14 www.visionias.in ©Vision IAS
From the table given above, it can be concluded that statement 1 is not correct.

Q 44.A
 Statement 1 is correct: EMISAT is an advanced electronic intelligence satellite jointly developed by
ISRO-DRDO. It is meant for electromagnetic spectrum measurements. It is modelled after a famous
Israeli spy satellite called SARAL (Satellite with ARgos and ALtika). EMISAT also has a special
altimeter (a radar altitude measuring device) called ‗AltiKa‘ that works in the Ka-band microwave region
of the spectrum. The electronic surveillance payload of EMISAT was developed under a DRDO‘s project
called KAUTILYA.It will help in intercepting signals broadcasted by communication systems, radars, and
other electronic systems. The Ka-band frequency allows EMISAT to scan through ice, rain, coastal zones,
land masses, forests and wave heights with ease.
 Statement 2 is not correct: India's PSLV-C45 was used to successfully inject EMISAT and 28
international customer satellites into their designated orbits on April 01, 2019 from Satish Dhawan Space
Centre SHAR, Sriharikota. This flight marked the first mission of PSLV-QL, a new variant of PSLV with
four strap-on motors.
 Statement 3 is not correct: It was launched to sun-synchronous polar orbit of 748 km height.
 It was also the first mission of ISRO where it launched satellites in three different orbits.

Q 45.D
 In the Gupta period, the crafts greatly improved in terms of quality as well as variety.
 Rust-proof iron and copper alloys were found and worked into intricate articles for civilian as well
as military purposes.
o The quality of the articles was so good that they were widely exported, even as far as Africa.
o In the design of these articles, there was, to an extent, Greco-Roman and Central Asian influence.
However, on the whole, they had a local character.
 In weaving, techniques were perfected for the making of cotton and silk materials.
o Manufacture of dyes and their widespread use in colouring textiles came into practice.
o Indian textile materials, especially from Varanasi and Bengal became famous for their light weight
and fine texture.
o The textiles became popular in the West and became an important commodity for export and trade.
 Opening up of previously inaccessible and uninhabited regions, organisation of better transport,
communication and trade routes helped the growth of trade.
o Internal trade was augmented by rapid development of foreign trade.
o Improvement in navigation by the Indians, especially using the knowledge of monsoons, and a
new design of seaworthy ships played an important role in this.
o The Indians traded with Arabs, the Mediterranean countries, especially Rome, Africa, south-east
Asian countries such as Java, Sumatra and Sri Lanka.

Q 46.A
 An important feature of the Chola administration was the local administration at districts, towns and
villages level.

15 www.visionias.in ©Vision IAS


 The Uttaramerur inscriptions in the modern Kanchipuram district speak much about the Chola
administration. Village autonomy was the most unique feature of the Chola administrative system.
 The lowest unit of the Chola administration was the village. The entire responsibility of the
village administration was in the hands of the village assembly.
 The village assemblies looked after the maintenance of peace, tanks, roads, public ponds, revenue
collection, judiciary, education and temples. The village assemblies were in charge of the payment of
taxes due from the villages to the treasury. They regulated public markets and helped people in times of
famine and flood. Assemblies provided provisions for education. They maintained law and order in every
village.
 There were thirty wards in each village. A representative for each ward was elected through
Kudavolai system. Each ward prepared a list of persons qualified for election to the various committees.
There were qualifications for a candidate standing for election to different committees.
o The candidate for instance, needed minimum educational qualification, had to be between 35 and 70
years of age, own landed property, have a house built in his own land, and be a taxpayer, etc.
 The Uttramerur inscriptions depict details about the method of electing members to the various
committees.
o In the Kudavolai system, the names of suitable candidates from a ward were written on palm leaf
tickets and put into a mud pot and placed before the general body. After shaking them well, a small
boy was asked to choose a ticket and give it to the arbitrator and he read out the name which was
again read out by priest. In this fashion thirty members for wards were elected.

Q 47.C
 The Constitution (Article 165) has provided for the office of the advocate general for the states. He
is the highest law officer in the state. Thus he corresponds to the Attorney General of India.
 The advocate general is appointed by the governor. He must be a person who is qualified to be
appointed a judge of a high court. In other words, he must be a citizen of India and must have held a
judicial office for ten years or been an advocate of a high court for ten years.
 The term of office of the advocate general is not fixed by the Constitution. Further, the Constitution does
not contain the procedure and grounds for his removal. He holds office during the pleasure of the
governor. This means that he may be removed by the governor at any time. He may also quit his office by
submitting his resignation to the governor. Conventionally, he resigns when the government (council of
ministers) resigns or is replaced, as he is appointed on its advice.
 As the chief law officer of the government in the state, the duties of the advocate general include the
following:
o To give advice to the government of the state upon such legal matters which are referred to him by the
governor.
o To perform such other duties of a legal character that are assigned to him by the governor.
o To discharge the functions conferred on him by the Constitution or any other law.
 In the performance of his official duties, the advocate general is entitled to appear before any court of law
within the state. Further, he has the right to speak and to take part in the proceedings of both the
Houses of the state legislature or any committee of the state legislature of which he may be named a
member, but without a right to vote. He enjoys all the privileges and immunities that are available to a
member of the state legislature.

Q 48.D
 Mohiniyattam as seen today has evolved through a long process of evolution. It traces its origin to the
temples of Kerala.
 Though the exact period of its origin is not known, there are evidences to prove the existence of a
community of female temple dancers who assisted the temple rituals by adding expressive gestures to the
mantras chanted by the temple priests.
 The dancers were called by different names during different periods of time.
 They were called as TaiNangai or Nangachi (one with beautiful hand), Dasi (servant), Tevitichi or
Deva-Adi-Achi (the one who served at the feet of the Lord), Koothachi (who performed koothu or dance).
 Their dances were known as ‗Nangai Natakam, Dasiyattam, Tevitichiyattam, etc.
 The Nangiars, who are the women folk of Nambiar community, still follow a strict code of dance,
performed in a small performing space, within the temple precincts, as practised in the olden times.
16 www.visionias.in ©Vision IAS
Q 49.B
 Article 110 of the Constitution deals with the definition of money bills. The Constitution lays down a
special procedure for the passing of money bills in the Parliament. A money bill can only be introduced in
the Lok Sabha and that too on the recommendation of the president. Every such bill is considered to be a
government bill and can be introduced only by a minister. Hence, option (a) is correct.
 After a money bill is passed by the Lok Sabha, it is transmitted to the Rajya Sabha for its consideration.
The Rajya Sabha has restricted powers with regard to a money bill. It cannot reject or amend a money
bill. It can only make the recommendations. It must return the bill to the Lok Sabha within 14 days,
wither with or without recommendations. The Lok Sabha can either accept or reject all or any of
the recommendations of the Rajya Sabha. Hence options (c) and (d) are correct.
 The Finance Bill is introduced to give effect to the financial proposals of the Government of India for the
following year. It is subjected to all the conditions applicable to a Money Bill. Unlike the Appropriation
Bill, the amendments (seeking to reject or reduce a tax) can be moved in the case of finance
bill. According to the Provisional Collection of Taxes Act of 1931, the Finance Bill must be enacted
(i.e., passed by the Parliament and assented to by the president) within 75 days. The Finance Act
legalises the income side of the budget and completes the process of the enactment of the budget. Hence
option (b) is not correct.

Q 50.D
 TB kills an estimated 480,000 Indians every year and more than 1,400 every day. India has the highest
burden of Tuberculosis and multi-drug-resistant TB (MDR-TB) in the world, disproportionately
high even for India‘s population. Recent evidences indicate that India‘s TB burden may be reducing, but
only very slowly.
 The National Strategic Plan for TB Elimination (NSP 2017-25) was launched by the government to
attain the vision of a TB-free India.
 Vision: TB-Free India with zero deaths, disease and poverty due to tuberculosis
 Goal: To achieve a rapid decline in burden of TB, morbidity and mortality while working
towardselimination of TB in India by 2025.
 Nikshay portal has been developed to monitor and track services and status related to Tuberculosis cases.
 DOTS or Directly Observed Treatment Short course is the internationally recommended strategy
for TB control that has been recognized as a highly efficient and cost-effective strategy.

Q 51.A
 Recently, Kerala witnessed their worst flood since 1924.
 Some of the Reasons for Kerela floods include: Incessant rainfall; Dam Mismanagement (sudden releases
of water from the Mullaperiyar dam - located in Kerala, but operated by Tamil Nadu); Stone quarrying,
Deforestation, Uncontrolled sand mining; Large expanse of low-lying areas etc.
 Operation ―Madad‖ by the Southern Naval Command (SNC) and Operation Sahayog by Army was
launched for assisting Kerela‘s administration in undertaking disaster relief.

Q 52.A
 Statement 1 is correct: The Financial Sector Assessment Program (FSAP) is a joint program of
the International Monetary Fund and the World Bank.
 Statement 2 is not correct: It was launched in 1999 in the wake of the Asian financial crisis. The
program brings together Bank and Fund expertise to help countries reduce the likelihood and severity of
financial sector crises. The FSAP provides a comprehensive framework through which assessors and
authorities in participating countries can identify financial system vulnerabilities and develop appropriate
policy responses.
 The program also helps bring financial sector analysis closer to the center of economic policy discussions
within a country and with the Fund and the Bank. Participating in the program helps inform domestic
policy-makers of the need for sequenced actions in areas requiring urgent attention and offers countries a
comprehensive framework in which to take on financial sector reforms. It also provides countries with an
opportunity to measure their compliance with financial sector standards and codes and, therefore, to
benchmark their regulatory and supervisory systems against internationally-accepted practices.
 The program also represents a collaborative international effort. Staff undertaking the assessments are
drawn from the World Bank and International Monetary Fund, as well as more than 50 official
institutions, central banks and supervisory agencies, from around the world. The international bodies
17 www.visionias.in ©Vision IAS
responsible for developing relevant financial sector standards also work closely with Fund and Bank staff
in implementing this program.
 The FSAP follows a three-pronged approach when looking at the country‘s financial sector:
o The soundness of a financial system versus its vulnerabilities and risks that increase the likelihood or
potential severity of financial sector crises.
o A country‘s developmental needs in terms of infrastructure, institutions and markets.
o A country‘s compliance with the observance of selected financial sector standards and codes.

Q 53.A
 As per provisions, Electoral Bonds may be purchased by a person, who is a citizen of India or
incorporated or established in India. A person being an individual can buy Electoral Bonds, either singly
or jointly with other individuals. Only the Political Parties registered under Section 29A of the
Representation of the People Act, 1951 (43 of 1951) and which secured not less than one per cent of the
votes polled in the last General Election to the House of the People or the Legislative Assembly of the
State, shall be eligible to receive the Electoral Bonds. Hence statement 1 is correct.
 Statement 2 is not correct: The Electoral Bonds shall be encashed by an eligible Political Party only
through a Bank account with the Authorized Bank.

Q 54.C
 Statement 2 is correct: HOPE - Healthcare Organizations‘ Platform for Entry-Level-Certification is a
portal for Entry-Level Certification Process with a focus to promote quality at nascent stages by enrolling
a wide range of hospitals across the country including Healthcare Organizations (HCOs) and Small
Healthcare Organizations (SHCOs). It will make Entry-Level Certification Process simpler, digital, faster
and user-friendly.
 It is an online platform for smooth and secure registration which provides a self-explanatory questionnaire
to be filled by the HCO/SHCOs. HOPE also enables them to comply with quality protocols, improve
patient safety and the overall healthcare facility of the organization.
 NABH is the implementing agency which is a constituent body of QCI (Quality Council of India (QCI)).
 Statement 1 is correct: The aim is to create a momentum for HCOs and SHCOs that want to avail
benefits associated with Insurance Regulatory and Development Authority of India (IRDAI) and
Ayushman Bharat by getting themselves NABH certified along with the primary aim of creating a quality
healthcare ecosystem in India. It will promote the idea of cashless payment to patients under insurance
coverage. T
 About NABH: NABH, a constituent body of QCI, has been working to ensure reliability, efficiency and
global accreditation in Indian healthcare sector.It uses contemporary methodologies and tools, standards
of patient safety and infection control.
 About QCI- Established in 1997 Quality Council of India (QCI) is an autonomous organization under the
DPIIT, MINISTRY OF COMMERCE AND INDUSTRY.It is the Quality Apex and National
Accreditation Body for accreditation and quality promotion in the country.The Council was established to
provide a credible, reliable mechanism for third party assessment of products, services and processes
which is accepted and recognized globally.

Q 55.D
 Central Water Commission (CWC) has been mandated to carry the Hydrological data observation on a
national basis.
 CWC has a network of 878 sites on major rivers and its tributaries of India. In addition to this, It has a
network of 76 exclusive meteorological stations.
 Items of observations & their frequency:
o Gauge( Water Level):- Gauges are being observed three times a day ( 8.00, 13.00, 18.00 hrs) during
non-monsoon period and hourly during the monsoon period.
o Discharge: Discharge is being observed once a day at 8:00 AM except on holidays.
o Silt: Silt samples are collected daily along with discharge observation. Thereafter, the samples are
being analyzed in the field laboratories (for coarse, medium and fine sediment).
o Bed Material: Bed material analysis is being done at 14 sites. The frequency of observation is thrice a
year.
o Water Quality:
 Base Station: One sample is being collected every two months and a total of six samples in a year.

18 www.visionias.in ©Vision IAS


 Trend Stations: Sample is being collected once in every month.
 Flux Stations: Samples is being collected thrice in a month,
 Trace and toxic metal are being analyzed twice in a year.
 Rainfall: Rainfall are being observed twice daily (8:00 AM and 8:00 PM)
Temperatures: Daily maximum and minimum temperatures are recorded at each site.
 Hence, all the given parameters are observed by CWC.

Q 56.B
 Since its inception in 1950 and till 15 October 1989, the election commission functioned as a single
member body consisting of the Chief Election Commissioner. On 16 October 1989, the president
appointed two more election commissioners to cope with the increased work of the election commission
on account of lowering of the voting age from 21 to 18 years. Thereafter, the Election Commission
functioned as a multimember body consisting of three election commissioners. However, the two posts of
election commissioners were abolished in January 1990 and the Election Commission was reverted to the
earlier position. Again in October 1993, the president appointed two more election commissioners. Since
then and till today, the Election Commission has been functioning as a multi-member body consisting of
three election commissioners.
 The chief election commissioner and the two other election commissioners have equal powers and
receive equal salary, allowances and other perquisites, which are similar to those of a judge of the
Supreme Court. In case of difference of opinion amongst the Chief election commissioner and/or
two other election commissioners, the matter is decided by the Commission by majority. Hence
statement 1 is not correct and statement 2 is correct.

Q 57.B
 The first team of unarmed military observers, who eventually formed the nucleus of the United Nations
Military Observer Group in India and Pakistan (UNMOGIP), arrived in the mission area in January
1949 to supervise, in the State of Jammu and Kashmir, the ceasefire between India and Pakistan,
and to assist the Military Adviser to the United Nations Commission for India and Pakistan (UNCIP),
established in 1948 by Security Council resolutions 39 and 47.
 Following the India-Pakistan hostilities at the end of 1971 and a subsequent ceasefire agreement of 17
December of that year, the tasks of UNMOGIP have been to observe, to the extent possible, developments
pertaining to the strict observance of the ceasefire of 17 December 1971 and to report thereon to the
Secretary-General.
 In July 1972, India and Pakistan signed an agreement defining a Line of Control in Kashmir which, with
minor deviations, followed the same course as the ceasefire line established by the Karachi Agreement in
1949. India took the position that the mandate of UNMOGIP had lapsed, since it related specifically to the
ceasefire line under the Karachi Agreement. Pakistan, however, did not accept this position.
 Given the disagreement between India and Pakistan about UNMOGIP‘s mandate and functions, the
Secretary-General‘s position has been that UNMOGIP can only be terminated by a decision of the
Security Council. In the absence of such a decision, UNMOGIP has been maintained with those same
arrangements since then.
 To fulfill UNMOGIP mandate, military observers conduct field tasks (area recce, field trip, field visit and
observation post) along the Line of Control. As part of the 1949 Karachi Agreement, UNMOGIP also
conducts investigations into alleged ceasefire violation complaints, which the two parties can submit to
the Mission. The findings of the investigations are shared with the Secretary-General and a summary of
investigations with the two parties.
 Hence, statement 1 is not correct and statement 2 is correct.

Q 58.D
 In the Minerva Mills case (1980), the Supreme Court held that ‗the Indian Constitution is founded on
the bedrock of the balance between the Fundamental Rights and the Directive Principles‘. Hence
pair 1 is correctly matched.
 In a significant judgement delivered in I.R. Coelho case (2007), the Supreme Court ruled that there could
not be any blanket immunity from judicial review of laws included in the Ninth Schedule. The court
held that judicial review is a ‗basic feature‘ of the constitution and it could not be taken away by putting a
law under the Ninth Schedule. Hence pair 2 is correctly matched.

19 www.visionias.in ©Vision IAS


 While upholding the Constitutional validity of the Tenth Schedule (Anti - Defection law), a Constitution
Bench of the Supreme Court in Kihoto Hollohan vs Zachillhu and Others declared that the Speaker‘s
decision in diqualifying a legislator is subject to judicial review as he acted as a tribunal while deciding
cases under the anti-defection law. Hence pair 3 is correctly matched.

Q 59.C
 Statements 1 and 2 are correct: Solar Park scheme - Launched by the Solar Energy Corporation of India
(SECI) to encourage the construction of solar parks that can generate electricity between 500 MW and
1000 MW.
 SECI is a Central PSU under the administrative control of the Ministry of New and Renewable Energy
(MNRE), set up on 20th Sept, 2011 to facilitate the implementation of JNNSM and achievement of targets
set therein. It is the only CPSU dedicated to the solar energy sector.

Q 60.A
 Right against Exploitation includes:
o Prohibition of traffic in human beings and forced labour, provided under Article 23 of the
constitution, and
o Prohibition of employment of children in factories, etc., provided under Article 24 of the constitution
 These fundamental rights are available to both citizens and non-citizens.
o Article 23 prohibits traffic in human beings, begar (forced labour) and other similar forms of forced
labour. Any contravention of this provision shall be an offence punishable in accordance with law.
This right is available to both citizens and non-citizens. It protects the individual not only against the
State but also against private persons.
o The expression ‗traffic in human beings‘ include (a) selling and buying of men, women and children
like goods; (b) immoral traffic in women and children, including prostitution; (c) devadasis; and (d)
slavery.
o Article 23 also provides for an exception to this provision. It permits the State to impose compulsory
service for public purposes, as for example, military service or social service, for which it is not
bound to pay. However, in imposing such service, the State is not permitted to make any
discrimination on grounds only of religion, race, caste or class.
o Article 20 grants protection against arbitrary and excessive punishment to an accused person,
whether citizen or foreigner or legal person like a company or a corporation. It is envisaged
under the Right to freedom.

Q 61.A
 Recently, the Reserve Bank of India (RBI) has introduced a Scheme on voluntary transition of Urban
Cooperative Bank into Small Finance Bank based on the recommendations of High-Powered
Committee on Urban Cooperative Banks under Rama Subramaniam Gandhi.
 UCBs with a minimum net worth of Rs.500 million (Rs 50 crore) and maintaining Capital to
Risk (Weighted) Assets Ratio of 9% and above are eligible to apply for voluntary transition to SFB.
Hence statement 3 is not correct. The promoters shall incorporate a public limited company under the
Companies Act, 2013 having the word ‗bank‘ in its name after receiving the in-principle approval from
RBI.
 As per Banking Regulation Act, 1949 a primary co-operative bank (Urban Co-operative Bank or UCB)
means a co-operative society, other than a primary agricultural credit society, whose,
o Principal business is the transaction of banking business;
o Paid-up share capital and reserves are not less than one lakh of rupees; and
o Bye-laws of which do not permit admission of any other co-operative society as a member: Provided
that this sub-clause shall not apply to theadmission of a co-operative bank as a member by reason of
such co-operative bank subscribing to the share capital of such co-operative society out of
funds provided by the State Government for the purpose.
 The UCBs are registered as cooperative societies under the provisions of, either the State
Cooperative Societies Act of the concerned State or the Multi State Cooperative Societies Act, 2002.
The Reserve Bank regulates and supervises the banking functions of UCBs under the provisions of
Banking Regulation Act, 1949 (As Applicable to Cooperative Societies). Hence both statement 1 and
2 are correct.
20 www.visionias.in ©Vision IAS
Q 62.C
 The granting of titles to men of merit was an important aspect of Mughal polity. A man‘s ascent in the
court hierarchy could be traced through the titles he held.
 The title Asaf Khan for one of the highest ministers originated with Asaf, the legendary minister of the
prophet king Sulaiman(Solomon).
 The title Mirza Raja was accorded by Aurangzeb to his two highest-ranking nobles, Jai Singh and Jaswant
Singh.
 Titles could be earned or paid for. Mir Khan offered Rs. one lakh to Aurangzeb for the letter alif, that is
A, to be added to his name to make it Amir Khan.
 Other awards included the robe of honour (khilat), a garment once worn by the emperor and
imbued with his benediction. One gift, the sarapa (―head to foot‖), consisted of a tunic, a turban and
a sash (patka). Jewelled ornaments were often given as gifts by the emperor. The lotus blossom set
with jewels (padma murassa) was given only in exceptional circumstances.
 A courtier never approached the emperor empty handed: he offered either a small sum of money (nazr) or
a large amount (peshkash). In diplomatic relations, gifts were regarded as a sign of honour and respect.

Q 63.A
 Deltas are depositional landforms found at the mouth of a river where the river meets a body of water with
a lower velocity than the river (e.g. a lake or the sea). For a delta to develop, the body of water needs to be
relatively quiet with a low tidal range so that deposited sediment isn‘t washed away and has time to
accumulate.
 Deltas can be of many types. The primary types of delta are cuspate, estuarine, arcuate and bird‘s foot.
 Arcuate deltas (e.g. The Nile Delta, Egypt) are shaped like a triangle (which is where the term delta
comes from, the Greek letter delta Δ) and form when a river meets a sea with alternating current directions
that shape the delta so that it looks like a triangle.
 Cuspate deltas (e.g. Ebro Delta, Spain) are vaguely shaped like a V with curved sides. Cuspate deltas
form when a river flows into a sea with waves that hit it head on, spreading the deposited sediment out.
 Bird‘s foot deltas (e.g. The Mississippi Delta) are shaped like (as the name suggests) a bird‘s foot. They
extend reasonably far into a body of water and form when the river‘s current is stronger than the sea‘s
waves. Bird‘s foot deltas are uncommon because there are very few areas where a sea‘s waves are weaker
than a river‘s current.
 Estuarine deltas form as a river does not empty directly into the ocean, but instead forms an estuary. An
estuary is a partly enclosed wetland that features a brackish water (part-saltwater, part-freshwater) habitat.
The Yellow River forms an estuary, for instance, as it reaches the Bohai Sea off the coast of northern
China.

Q 64.A
 For administration of Investor Education and Protection Fund Government of India has on 7th September,
2016 established Investor Education and Protection Fund Authority under the provisions of section 125 of
the Companies Act, 2013. Hence statement 1 is correct.
 The Authority is entrusted with the responsibility of administration of the Investor Education Protection
Fund (IEPF), make refunds of shares, unclaimed dividends, matured deposits/debentures etc. to investors
and to promote awareness among investors. Hence statement 2 is correct.
 The secretary to the Ministry of Corporate Affairs acts as the ex-officio Chairman of the Authority. Hence
statement 3 is not correct.

Q 65.C
 Statement 1 is correct: Body mass index (BMI) is a measure of body fat based on height and weight.
It is a measure for indicating nutritional status in adults. It is defined as a person‘s weight in kilograms
divided by the square of the person‘s height in metres (kg/m2). For example, an adult who weighs 70 kg
and whose height is 1.75 m will have a BMI of 22.9.
 Statement 2 is not correct: For most adults, an ideal BMI is in the 18.5 to 24.9 range. For children
and young people aged 2 to 18, the BMI calculation takes into account age and gender as well as height
and weight.
 If your BMI is:
o below 18.5 – you're in the underweight range
o between 18.5 and 24.9 – you're in the healthy weight range
21 www.visionias.in ©Vision IAS
o between 25 and 29.9 – you're in the overweight range
o between 30 and 39.9 – you're in the obese range
 Statement 3 is correct: The correlation between the BMI and body fatness is fairly strong, but even
if 2 people have the same BMI, their level of body fatness may differ. In general,
 At the same BMI, women tend to have more body fat than men.
 At the same BMI, Blacks have less body fat than do Whites, and Asians have more body fat than do
Whites.
 At the same BMI, older people, on average, tend to have more body fat than younger adults.
 At the same BMI, athletes have less body fat than do non-athletes.

Q 66.C
 IMPacting Research, INnovation and Technology (IMPRINT), a national initiative of the Ministry of
Human Resource Development (MHRD) designed to address all major engineering challenges
relevant to India through an inclusive and sustainable mode of translational research has now reached its
next phase, IMPRINT Round 2.
 IMPRINT-2 has been approved by Government of India (GoI) with a revised strategy under which, this
national initiative will be jointly funded and steered by MHRD and Department of Science and
Technology (DST).
 While any faculty member from an MHRD funded Higher Education Institute (HEI) including Centrally
Funded Technical Institution (CFTI) may lead as the Principal Investigator (PI), single or multiple
partners from stake holder ministry, institutions, PSUs, strategic agencies and industry are welcome as
Co-PI, partner or collaborator.
 The Uchhatar Avishkar Yojana (UAY) scheme will be subsumed with IMPRINT-2 and no further
call for proposal submission under UAY will be issued.

Q 67.C
 A parasite is an organism that lives on or in a host organism and gets its food from or at the expense of its
host.
 There are three main classes of parasites that can cause disease in humans:
o Protozoa
o Helminths
o Ectoparasites
 Malaria is caused by protozoan parasites called Plasmodia
o Human malaria is caused by four different species of Plasmodium: P. falciparum, P. malariae, P.
ovale and P. vivax
 In humans, the parasites grow and multiply first in the liver cells and then in the red cells of the blood.
 In the blood, successive broods of parasites grow inside the red cells and destroy them, releasing daughter
parasites (―merozoites‖) that continue the cycle by invading other red cells.
 The blood stage parasites are those that cause the symptoms of malaria

Q 68.D
 SA node (SA stands for sinoatrial)
o It is one of the major elements in the cardiac conduction system, the system that controls the heart
rate. This system generates electrical impulses and conducts them throughout the muscle of the
heart, stimulating the heart to contract and pump blood.
o The SA node is the heart's natural pacemaker. The SA node consists of a cluster of cells that are
situated in the upper part of the wall of the right atrium (the right upper chamber of the heart). The
electrical impulses are generated there. The SA node is also called the sinus node.
 Pulmonary veins
o Veins are the blood vessels that carry blood to the heart.
o Pulmonary veins are responsible for carrying oxygenated blood from the lungs back to the left
atrium of the heart.
o This differentiates the pulmonary veins from other veins in the body, which are used to carry
deoxygenated blood from the rest of the body back to the heart.
o Humans have four pulmonary veins in total, two from each lung
 Venae cavae
o vena cava are two large veins that return deoxygenated blood from the body into the heart.

22 www.visionias.in ©Vision IAS


o In humans, there are the superior vena cava and the inferior vena cava
o The superior vena cava is the large vein which returns blood to the heart from the head, neck and both
upper limbs.
o The inferior vena cava returns blood to the heart from the lower part of the body.
Hence, all pairs are correctly matched.

Q 69.C
 Baghuz is a village in Eastern Syria, which was the last 1.5-square-mile remnant of the ISIS' territory in
the region. In March 2019, the village was finally retaken, erasing the militant theocracy that once
spanned two countries. In the battle, the Syrian Democratic Forces backed by USA played a pivotal role
to eradicate ISIS. The Baghuz militants waged a surprisingly fierce defense and kept the American-
backed coalition at bay for months.

Q 70.A
 Recapitalisation of public sector banks (PSBs), the ongoing improvement in their financials, and
resolution of stressed assets under the Insolvency and Bankruptcy Code are expected to improve bank
credit offtake, according to Reserve Bank of India‘s latest monetary policy report. Credit Offtake is the
credit portfolio of the banks. It signifies the credit given by the banks.
 Increase in Credit Off take is an indication that Economy is recovering and purchasing power of the
people is increasing resulting in increase in demand of various products. To meet with the increased
demand, Trade & Industry and other sectors withdraw more and more funds from Banks.

Q 71.A
 The term of a member is six years; but a member elected in a bye-election serves for the remainder of the
term of the vacancy caused. Hence statement 1 is correct.
 Where there are two or more parties in opposition to the Government, having the same numerical strength,
the Chairman, having regard to the status of the parties, recognizes anyone of the Leaders of such parties
as the Leader of the Opposition and such recognition is final and conclusive. Hence statement 2 is not
correct.

Q 72.D
 Electors who will not be able to produce the EPIC can produce any of the 11 alternative photo identity
documents for establishing their identity. These are passport, driving license, service identity cards
with photograph issued to employees by the central or the state government or PSUs or public
limited companies, passbooks with photograph issued by a bank or a post office, PAN Card, smart
card issued by the Registrar General of India (RGI) under National Population Register (NPR),
MGNREGA job card, health insurance smart card issued under the scheme of Ministry of Labour,
pension document with photograph, official identity cards issued to MPs/ MLAs/ MLCs and
Aadhaar Card.
 In the case of EPIC, a clerical error or spelling mistakes should be ignored provided the identity of the
elector can be established by the EPIC. If it is not possible to establish the identity of the elector on
account of mismatch of photograph, the voter shall have to produce one of the above alternative photo
documents.

Q 73.B
 Mahi cuts across Tropic of cancer twice. Mahi River is the only river in India that cuts the Tropic of
Cancer twice, first in Madhya Pradesh from where it flows towards Rajasthan and enters Gujarat where it
cuts for the second time.
 Damodar and Chambal cut across Tropic of Cancer while Tapi, which flows majorly in Maharashtra

does not cut across the tropic of cancer.

Q 74.A
 In the Kesavananda Bharati case (1973), Supreme Court laid down a new doctrine of the ‗basic structure‘
(or ‗basic features‘) of the Constitution. It ruled that the constituent power of Parliament under
Article 368 does not enable it to alter the ‗basic structure‘ of the Constitution. Hence, statement 1 is
correct.
23 www.visionias.in ©Vision IAS
 The present position is that the Parliament under Article 368 can amend any part of the Constitution
including the Fundamental Rights but without affecting the ‗basic structure‘ of the
Constitution. However Basic structure is not mentioned in the Constitution. From the various
judgements of the Supreme Court, few important features have emerged as ‗basic features‘ of the
Constitution such as Federal character of the Constitution, Secular character of the Constitution
etc. Hence statement 2 is not correct.
 In the Waman Rao case 5 (1981), the Supreme Court adhered to the doctrine of the ‗basic structure‘ and
further clarified that it would apply to constitutional amendments enacted after April 24, 1973 (i.e.,
the date of the judgement in the Kesavananda Bharati case). Hence statement 3 is not correct.

Q 75.C
 Mahatma Gandhi had launched the Champaran satyagraha in 1917 to address the issues facing local
farmers who were reeling from the oppressive tinkathia system under which indigo planters used to force
them to grow indigo on certain portions of their land.
 This indigo was used to make dye. But the demand for indigo dropped when the Germans invented a
cheaper artificial dye. However, during the First World War the German dye ceased to be available and
indigo once more became profitable for the British. Many tenants were forced again into indigo
cultivation --required by their lease under British law.
 This led to anger and resentment among the tenants. A farmer named Raj Kumar Shukla appealed to
Gandhiji to organize the struggle to save the plantation workers, and Gandhiji visited the district in April
1917.
o Gandhiji‘s arrival unsettled the British and he was ordered to leave the district. When Gandhiji
refused, he was arrested by the police on the charge of creating unrest, leading to massive non-violent
protests and rallies, which eventually led to his release.
o The protests and strikes against the landlords eventually came to an end and the landlords signed an
agreement, with the guidance of the British government, granting farmers more compensation and
control over farming, along with other benefits.
 Development of Gandhi Circuit in Bihar is one of the projects identified for development under the
Special Package for Bihar announced in 2015. A project for ―Development of Gandhi Circuit:
Bhitiharwa-Chandrahia-Turkaulia under Rural Circuit theme of Swadesh Darshan Scheme‖ has
been sanctioned in Bihar with central financial assistance of Rs. 4465.02 lakh.
o The Bhitiharwa Ashram is located in Gaunaha block of West Champaran, around 200km northwest
of Patna, and it used to be the centre of activity during the Champaran satygarha.
o Chandrahia is a place located on the outskirts of Motihari town where Gandhiji had been stopped by
the local administration while he was going to visit a few villages in Champaran to interact with the
residents.
o Turkaulia, located around 15km southeast of Motihari, had witnessed a movement of local farmers
against indigo planters.

Q 76.C
 Features of the Indian independence Act, 1947:
o It ended the British rule in India and declared India as an independent and sovereign state from
August 15,1947.
o It provided for the partition of India and creation of two independent dominions of India and Pakistan
with the right to secede from the British Commonwealth.
o It abolished the office of viceroy and provided, for each dominion, a governor-general, who was to be
appointed by the British King on the advice of the dominion cabinet. His Majesty‘s Government in
Britain was to have no responsibility with respect to the Government of India or Pakistan. Hence
statement 1 is correct.
o It empowered the Constituent Assemblies of the two dominions to frame and adopt any constitution
for their respective nations and to repeal any act of the British Parliament, including the Independence
act itself. Hence statement 2 is correct.
o It empowered the Constituent Assemblies of both the dominions to legislate for their respective
territories till the new constitutions were drafted and enforced. No Act of the British Parliament
passed after August 15, 1947 was to extend to either of the new dominions unless it was extended
thereto by a law of the legislature of the dominion.

24 www.visionias.in ©Vision IAS


o It abolished the office of the secretary of state for India and transferred his functions to the secretary
of state for Commonwealth Affairs.
o It proclaimed the lapse of British paramountcy over the Indian princely states and treaty relations with
tribal areas from August 15,1947.
o It granted freedom to the Indian princely states either to join the Dominion of India or Dominion of
Pakistan or to remain independent.
o It provided for the governance of each of the dominions and the provinces by the Government of India
Act of 1935, till the new Constitutions were framed. The dominions were however authorised to make
modifications in the Act.
o It deprived the British Monarch of his right to veto bills or ask for reservation of certain bills for his
approval. But, this right was reserved for the Governor-General. The Governor-General would have
full power to assent to any bill in the name of His Majesty.
o It designated the Governor-General of India and the provincial governors as constitutional (nominal)
heads of the states. They were made to act on the advice of the respective council of ministers in all
matters.
o It dropped the title of Emperor of India from the royal titles of the king of England.
o It discontinued the appointment to civil services and reservation of posts by the secretary of state for
India. The members of the civil services appointed before August 15, 1947 would continue to enjoy
all benefits that they were entitled to till that time.

Q 77.C
 The President of India and the Governor of States enjoy the following immunities (Article 361):
o The President or the Governor is not answerable to any court for the exercise and performance of the
powers and duties of his office.
o No criminal proceedings shall be instituted or continued against the President or the Governor
in any court during his term of office.
o No process for the arrest or imprisonment of the President or the Governor shall be issued from any
court during his term of office.
o No civil proceedings against the President or the Governor shall be instituted during his term of office
in any court in respect of any act done by him in his personal capacity, whether before or after he
entered upon his office, until the expiration of two months next after notice has been delivered to
him.
 Parliamentary privileges are special rights, immunities and exemptions enjoyed by the two Houses of
Parliament, their committees and their members. They are necessary in order to secure the independence
and effectiveness of their actions. The Constitution has also extended the parliamentary privileges to those
persons who are entitled to speak and take part in the proceedings of a House of Parliament or any of its
committees. These include the attorney general of India and Union ministers. The parliamentary
privileges do not extend to the president although he is also an integral part of the Parliament.

Q 78.A
 Located in Palmapet in Mulugu district of Telangana, the Ramappa Temple, also known as the
Ramalingeshwara Temple, dates back to 1213 AD and was built by General Recherla Rudra, during the
period of the Kakatiya ruler Ganapati Deva. The temple, known for its beautiful intricate carvings, is the
only temple in the country which has been named after its sculptor, Ramappa, who took 40 years to build
the temple. The temple is an 800-year-old engineering marvel of the Kakatiya dynasty.Hence statement 1
and 2 are correct.
 The Ramappa Temple in Warangal, a Kakatiya era structure, has been nominated for the UNESCO World
Heritage site selection process for 2019. Furthermore, it is India‘s only entry this year. Hence statement 3
is not correct.

Q 79.B
 The Union Cabinet chaired by Prime Minister has approved the signing of Migration and Mobility
Partnership Agreement between India and France.
 The Agreement represents a major milestone in enhancing people-to-people contacts, fostering mobility
of students, academics, researchers and skilled professionals and strengthening cooperation on issues
related to irregular migration and human trafficking between the two sides. The Agreement is a testimony
to India's rapidly expanding multi-faceted relationship with France and symbolizes the increasing trust
25 www.visionias.in ©Vision IAS
and confidence between the two sides. The Agreement is initially valid for a period of seven years,
incorporates provision for automatic renewal and a monitoring mechanism through a Joint Working
Group.

Q 80.D
 The Green Highways Policy 2015 was launched with the vision to develop eco-friendly National
Highways with the participation of the community, farmers, NGOs, private sector, institutions,
government agencies and the Forest Department for economic growth and development in a sustainable
manner.
 Green Highways Division (erstwhile National Green Highways Mission NGHM) under National
Highways Authority of India (Ministry of Road Transport & Highways) has been entrusted with the task
of planning, implementation and monitoring roadside plantations along one lakh km network of National
Highways which would in turn generate one lakh direct employment opportunity in plantations sector in
next ten years.
 Statements 1 and 2 are correct: 1% of the total project cost of all highways projects will be kept aside
for the highway plantation and its maintenance. About Rs. 1000 crore per year will be available for
plantation purpose. This policy will generate employment opportunities for about five lakh people from
rural areas.
 Statement 3 is correct: There will be strong monitoring mechanism in place by using ISRO‘s Bhuvan
and GAGAN satellite systems. Every planted tree will be counted and auditing will be done.

Q 81.A
 Stars twinkle because of turbulence in the atmosphere of the Earth. As the atmosphere churns, the light
from the star is refracted in different directions. This causes the star's image to change slightly in
brightness and position, hence "twinkle."
o Refraction is the change in direction of a wave caused by a change in speed as the wave passes from
one medium to another. Snell's law explains this change.
 Rainbow formation involves a series of physical phenomena - reflection, refraction, dispersion and total
internal reflection.
 The occurrence of each of these is due to the interaction of light with air and water and the boundaries
between them.
 Mirage is an optical phenomenon that creates the illusion of water and results from the refraction of light
through a non-uniform medium. Mirages are most commonly observed on sunny days when driving down
a roadway.
o Its formation is a result of the refraction and the total internal reflection of light in the air.
o Total internal reflection is the reflection of the total amount of incident light at the boundary between
two media.

Q 82.B
 During colonial period (British period) millions of the indentured labourers were sent to Mauritius,
Caribbean islands (Trinidad, Tobago and Guyana), Fiji and South Africa by British from Uttar Pradesh
and Bihar; to Reunion Island, Guadeloupe, Martinique and Surinam by French and Dutch and by
Portuguese from Goa, Daman and Diu to Angola, Mozambique to work as plantation workers. All such
migrations were covered under the time-bound contract known as Girmit Act (Indian Emigration Act).
However, the living conditions of these indentured labourers were not better than the slaves. These
people were called Girmitiyas.

Q 83.C
 Committee on Private Members‘ Bills and Resolutions: This committee classifies bills and allocates time
for the discussion on bills and resolutions introduced by private members (other than ministers). This is a
special committee of the Lok Sabha and consists of 15 members, including the Deputy Speaker as its
chairman. Hence statements 1 and 2 are correct.
 The Rajya Sabha does not have any such committee. The same function in the Rajya Sabha is performed
by the Business Advisory Committee of that House.

26 www.visionias.in ©Vision IAS


Q 84.A
 The Global Atmosphere Watch (GAW) programme of WMO is a partnership involving the Members
of WMO, contributing networks and collaborating organizations and bodies which provides reliable
scientific data and information on the chemical composition of the atmosphere, its natural and
anthropogenic change, and helps to improve the understanding of interactions between the
atmosphere, the oceans and the biosphere.
 GAW focal areas are aerosols, greenhouse gases, selected reactive gases, ozone, UV radiation and
precipitation chemistry (or atmospheric deposition).
 The Atmospheric Environment and Research Division of WMO‘s Research Department publishes
the WMO-GAW Annual Greenhouse Gas Bulletins. Each year, these bulletins will report the latest
trends and atmospheric burdens of the most influential, long-lived greenhouse gases; carbon dioxide
(CO2), methane (CH4), and nitrous oxide (N2O), as well as a summary of the contributions of the lesser
gases.

Q 85.B
 Glacial Lakes Outburst Floods (GLOFS): Floods caused due to outburst of glacial lakes is known as
GLOF. The moraine wall act as a natural dam, trapping the melt water from the glacier and leading to the
formation of a glacial lake.
 Factors triggering GLOFS
o Retreat of glaciers and change in radiative balance in the region in the wake of global warming.
o Increasingly erratic and unpredictable monsoon rainfall patterns and increased climate variability.
o Anthropogenic activities such as mass tourism; developmental interventions such as roads and
hydropower projects; and the practice of slash and burn type of farming in certain pockets of the
Indian Himalayan region
o Black carbon also plays important factor which melts the ice on the mountain due to albedo effect.
o Other Factors like cascading processes (flood from a lake situated upstream), earthquake, blocking
of subsurface outflow tunnels, long term dam degradation also trigger GLOFS.

Q 86.B
 Kandukuri Veeresalingam was one of the reformists from Andhra, who gave great moral courage and
showed the way to the Andhra society.
 He is the man who first brought the Renaissance in Andhra, and considered as the prophet of Modern
Andhra.
 He was deeply influenced by the ideals and activities of ‗Brahma Samaj‘ and particularly those of
Atmuri Lakshmi Narasimham.
 He fought against social customs like child marriage, caste-system, prostitution etc. He fought for the
upliftment of young widows education for women, and remarriage of widows.
 Because of these reformist activities, he was highly appreciated by the great social reformers like
M.G.Ranade, Iswar Chandra Vidyasagar and D.K.Karve.
 The Government, in appreciation of his work, conferred on him the title of "Rao Bahadur" in 1893.
 He was acclaimed as ‗Iswar Chandra Vidyasagar of the south‘ by the famous reformer M.G.Ranade at
a meeting in 1898

Q 87.A
 Statement 1 is correct: The Green Growth Knowledge Platform (GGKP) is a global network of
international organizations and experts that identifies and addresses major knowledge gaps in green
growth theory and practice. By encouraging widespread collaboration and world-class research. GGKP
offers practitioners and policymakers the policy guidance, good practices, tools, and data necessary to
support the transition to a green economy.
 The initiative is led by the Global Green Growth Institute, the Organisation for Economic Co-
operation and Development, the United Nations Environment Programme, United Nations
Industrial Development Organization, and the World Bank.
 This group has since expanded to include a large, diverse group of knowledge partners, comprising
leading institutions and organizations active in areas related to green growth and green economy at the
local, national, regional, and international levels.The GGKP benefits from strategic funding from
Switzerland, as well as programmatic funding from the Netherlands, Germany, and the MAVA
Foundation.
27 www.visionias.in ©Vision IAS
 Statement 2 is incorrect: The GGKP was established in January 2012, much before the Katowice
summit which happened in 2018.

Q 88.D
 The basic structure of Indian constitution stands on the Government of India Act, 1935. Constitutions of
other major countries were studied and some features of their constitution have been added in our
constitution.
 Features that have been borrowed:
o British Constitution: Parliamentary government, Rule of Law, legislative procedure, single
citizenship, cabinet system, prerogative writs, parliamentary privileges and bicameralism. Hence
pair 1 is correctly matched.
o Government of India Act, 1935: Federal Scheme, Office of governor, Judiciary, Public Service
Commissions, Emergency provisions and administrative details.
o US Constitution: Fundamental rights, independence of judiciary, judicial review, impeachment of the
president, removal of Constitution Supreme Court and high court judges and post of vice-president
o Irish Constitution: Directive Principles of State Policy, nomination of members to Rajya Sabha
and method of election of President. Hence pair 3 is correctly matched.
o Canadian Constitution: Federation with a strong Centre, vesting of residuary powers in the Centre,
appointment of state governors by the Centre, and advisory jurisdiction of the Supreme Court.
o Australian Constitution: Concurrent List, freedom of trade, commerce and inter-course, and joint
sitting of the two Houses of Parliament. Hence pair 2 is correctly matched.

Q 89.B
 People, generally are emotionally attached to their place of birth. But millions of people leave their places
of birth and residence. There could be variety of reasons. These reasons can be put into two broad
categories : (i) push factor, these cause people to leave their place of residence or origin; and (ii) pull
factors, which attract the people from different places.
 In India people migrate from rural to urban areas mainly due to poverty, high population pressure on the
land, lack of basic infrastructural facilities like health care, education, etc. Apart from these factors,
natural disasters such as, flood, drought, cyclonic storms, earthquake, tsunami, wars and local conflicts
also give extra push to migrate.
 On the other hand, there are pull factors which attract people from rural areas to cities. The most
important pull factor for majority of the rural migrants to urban areas is the better
opportunities, availability of regular work and relatively higher wages. Better opportunities for
education, better health facilities and sources of entertainment, etc. are also quite important pull factors.

Q 90.A
 In conjunction with Government‘s initiative on ‗Start-Up India‘ and ‗Digital India‘, FSSAI (Food Safety
and Standards Authority of India) is bringing together innovators and start up entrepreneurs to
provide innovative solutions and transform country‘s food safety and nutrition landscape.
 FSSAI has identified four broad challenges that it is seeking to solve through this initiative:
o Cheaper, Rapid and Anywhere Food Testing
o Ensuring Availability of Healthy Food
o Educating India and Rethinking Food Labelling: Eat Safe, Eat Right and Be a Smart Consumer
o Save Food, Feed Hungry: Waste Food Recovery
 Through FINE, FSSAI is collaborating with Start-ups working to solve the listed four challenges.
 FSSAI's vision is to create a ecosystem of start-ups working to solve the same challenge and inter-connect
them. The objectve of defining and identifying challenges is to be able to create a platform that brings
together aspiring and established entrepreneurs in the food and nutrition space to transform the food and
nutrition landscape in the country.

Q 91.A
 Sweat equity refers to the effort and toil a company‘s owners and employees contribute to a project or
enterprise—and the value it creates. In cash-strapped startups, owners and employees typically accept
salaries that are below their market values in return for a stake in the company, which they hope to profit
from when the business is eventually sold.

28 www.visionias.in ©Vision IAS


 It is the non-monetary investment that owners or employees contribute to a business venture. Startups and
entrepreneurs often use this form of capital to fund their businesses by compensating their employees with
stock rather than cash, especially when cash is in short supply.
 The main benefit is that the Sweat equity allows companies to raise funds without raising debt levels.
In many situations where some members of a partnership are contributing their money and others are
spending time, the partnership may be composed of cash and non-cash (or sweat equity). Ultimately,
sweat equity is rewarded the same as cash equity through a distribution of stock or other forms of equity
in a start-up venture.

Q 92.D
 The UPSC presents, annually, to the president a report on its performance. The President places this
report before both the Houses of Parliament, along with a memorandum explaining the cases where the
advice of the Commission was not accepted and the reasons for such non-acceptance.
 The National Commission for SCs presents an annual report to the president. It can also submit a
report as and when it thinks necessary. The President places all such reports before the
Parliament, along with a memorandum explaining the action taken on the recommendations made by the
Commission. The memorandum should also contain the reasons for the non-acceptance of any of such
recommendations.
 The National Human Rights Commission submits its annual or special reports to the Central
government and to the state government concerned. These reports are laid before the respective
legislatures, along with a memorandum of action taken on the recommendations of the commission and
the reasons for non-acceptance of any of such recommendations.
 The Central Information Commission submits an annual report to the Central Government on the
implementation of the provisions of this Act. The Central Government places this report before
each House of Parliament.
 The CVC has to present annually to the President a report on its performance. The President places
this report before each House of Parliament.

Q 93.A
 An expert committee was constituted under Dr. Anupam Verma, Former Professor, Indian Agriculture
Research Institute (IARI), to carry out technical review of 66 pesticides that are banned, restricted,
withdrawn in one or more countries but continued to be registered in India.
 The Expert Committee, inter alia, recommended 13 pesticides to be banned, 27 pesticides to be reviewed
in 2018 after completion of certain technical studies and 6 pesticides to be phased out by 2020. The
Committee further recommended continuation of ban on 1 pesticide and did not offer any assessment of a
pesticide which is currently sub judice.
Q 94.B
 In December, 2017, India‘s first modern conventional submarine, INS Kalvari, has been commissioned
into Navy‘s fleet.
 The diesel-electric submarine is designed by French naval defence and energy company DCNS. The
submarine is named after the dreaded tiger shark, a deadly deep sea predator of the Indian Ocean.
 The submarine boasts of superior stealth features such as advanced acoustic silencing techniques, low
radiated noise levels, hydro-dynamically optimised shape and the ability to launch a crippling attack on
the enemy using precision-guided weapons.

Q 95.A
 In the year 1864, Keshub Chandra Sen, then a member of the Brahmo Samaj, visited Madras, in
the course of his missionary tour of India, and delivered lectures on Brahmoism.
 His visit was warmly welcomed and his lectures were eagerly listened to.
 Under the influence of these lectures which stimulated religious inquiry, the 'Veda Samaj' was
established in the same year.
 The leaders of the 'Veda Samaj' were V. Rajagopalacharyulu (President), aad P. Subbarayalu Chetty
(secretary), the two leading members of the Madras bar.
 The covenants of the Veda Samaj professed the worship of "the one only without a second" and vowed to
practise rituals agreeable to the spirit of pure Theism, and free from the superstitions and absurdities
which characterized Hindu ceremonies.
 The other covenants spoke about discarding of caste distinctions, boycotting nautch parties, promoting
remarriage of widows, encouraging female education, and liberally contributing to the publication of
tracts and prayer books in the vernaculars.

29 www.visionias.in ©Vision IAS


 As the members of the Samaj still held to the ancient sacred books, the Vedas and other sacred Hindu
literature, they called their Samaj by the name 'Veda Samaj.'
 Though this Samaj made a good beginning in the sphere of religious reform in the south, it was still an
immature attempt. The two prominent members of the Samaj mentioned above died in 1868 and after a
while the Samaj was crippled.
 Around 1869, Chembeti Sridharalu Naidu of Cuddalore became the secretary of the Veda Samaj. He had a
strong grasp of theistic principles, and, at length, succeeded in converting the society into the Brahmo
Samaj of Southern India in 1871.
 He prepared the new covenants in the true spirit of Brahmoism, which replaced the old ones of the Veda
Samaj.

Q 96.D
 Recently, SpaceX had launched India's first privately built satellite ExseedSAT 1 along with 63 other
satellites from 18 countries.
 The mini communication satellite weighing just a kg with double the size of a Rubik‘s cube (10 cm x 10
cm x 10 cm) is made up of aluminium alloy. The satellite, which is the brainchild of a Mumbai-based
start-up Exseed Space, looks to serve the amateur radio community. ExseedSAT 1 was built at a cost of
Rs 2 crore. The satellite with a lifespan of five years will allow people to receive signals on 145.9 Mhz
frequency with the help of a TV tuner. Built in just 18 months, ExseedSAT 1 has paved the way for
private-funded space missions.

Q 97.B
 When the British tried to annex the small state of Kitoor (in Karnataka today), Rani Channamma took
to arms and led an anti-British resistance movement. She had became queen of Kitoor when she
married Raja Mallasarja of the Desai family. They had one son who died in 1824.
 After the death of her son, she adopted another child, Shivalingappa, and made him heir to the throne.
However, the British ordered Rani Chennamma to exile the adopted child Shivalingappa, using the
policy of paramountcy and complete authority. But Chennamma defied the order.
 She was arrested in 1824 and died in prison in 1829. But Rayanna, a poor chowkidar of Sangoli in Kitoor,
carried on the resistance. With popular support he destroyed many British camps and records. He was
caught and hanged by the British in 1830.
 So, the princely state of Kitoor was taken over by the British East India Company even before the
'doctrine of lapse' was officially articulated by Lord Dalhousie, Governor General for the British East
India Company, between 1848 and 1856.
o The doctrine of lapse declared that if an Indian ruler died without a male heir his kingdom would
'lapse', that is, become part of Company territory. Lord Dalhousie made it official in 1848.
o Dalhousie's annexations and the 'doctrine of lapse' caused great anger among the ruling princes in
India, making it one of the causes of the Indian Mutiny of 1857.
 The Raja of Travancore had been asked by the Company officers to meet the entire expenditure of the
Third Anglo-Mysore war on the plea that the war was undertaken in defense of Travancore. The Raja was
forced to entertain a subsidiary force far beyond his capacity to subsidise. The Raja was forced to raise
loans from bankers and merchants.
 Velu Thampi, the newly appointed Dalava(Prime Minister) tried to put the State's finances in order by
reducing expenditure and increasing revenues wherever possible. This led to some unrest among the
troops.
 Alarmed, the Madras Government asked for a modification of an earlier treaty to enable the British
troops to be used to aid the Raja in quelling internal commotions as well. Thus a new treaty of
perpetual friendship and alliance was signed in January 1805.
 The new treaty was not well received, especially by Velu Thampi Dalava. The Dalava began concerted
moves for an open rebellion against the British in defense of the king and the country. He began to recruit
soldiers and collect arms.
 On January 16, 1809 Velu Thampi issued a historic proclamation at Kundara calling upon the
people to rise en masse against the British. The response was wide-spread and in many places
British troops were put in peril.
 But, as British contingents began to converge on Travancore from different directions, the rebels lost heart
and the revolt began to peter out. The Raja, who was anxious about the safety of his throne, wrote to the
Resident requesting for the cessation of hostilities.
 Peace was concluded in March 1809. Velu Thampi, who was hiding in the Mannadi Temple, committed
suicide.
30 www.visionias.in ©Vision IAS
Q 98.C
 The Intergovernmental Science-Policy Platform on Biodiversity and Ecosystem Services (IPBES) is an
independent intergovernmental body, established by member States in 2012. The objective of IPBES is to
strengthen the science-policy interface for biodiversity and ecosystem services for the conservation and
sustainable use of biodiversity, long-term human well-being and sustainable development. Its secretariat is
located in Bonn, Germany. Hence statement 1 is correct and statement 2 is not correct.
 IPBES currently has over 130 member States. A large number of NGOs, organizations, conventions and
civil society groupings also participate in the formal IPBES process as observers, with several thousand
individual stakeholders, ranging from scientific experts to representatives of academic and research
institutions, local communities and the private sector, contributing to and benefiting from our work.
 India is one of the member States of the IPBES. India joined IPBES on 21st April, 2012 on the same day
the IPBES was established. Hence statement 3 is correct.
Q 99.C
 An Initial Coin Offering (ICO) is the cryptocurrency space's rough equivalent to an IPO in the
mainstream investment world. ICOs act as fundraisers of sorts; a company looking to create a new coin,
app, or service launches an ICO. Next, interested investors buy in to the offering, either with fiat currency
or with preexisting digital tokens like ether. In exchange for their support, investors receive a new
cryptocurrency token specific to the ICO. Investors hope that the token will perform exceptionally well
into the future, providing them with a stellar return on investment. The company holding the ICO uses the
investor funds as a means of furthering its goals, launching its product, or starting its digital currency.
ICOs are used by startups to bypass the rigorous and regulated capital-raising process required by venture
capitalists or banks.
 When a cryptocurrency startup firm wants to raise money through an Initial Coin Offering (ICO), it
usually creates a plan on a whitepaper which states what the project is about, what need(s) the project will
fulfill upon completion, how much money is needed to undertake the venture, how much of the virtual
tokens the pioneers of the project will keep for themselves, what type of money is accepted, and how long
the ICO campaign will run for. During the ICO campaign, enthusiasts and supporters of the firm‘s
initiative buy some of the distributed cryptocoins with fiat or virtual currency. These coins are referred to
as tokens and are similar to shares of a company sold to investors in an IPO-type transaction. If the money
raised does not meet the minimum funds required by the firm, the money is returned to the backers and
the ICO is deemed to be unsuccessful. If the funds requirements are met within the specified timeframe,
the money raised is used to either initiate the new scheme or to complete it.

Q 100.A
 INAM-Pro+ is an upgraded version of INAM-Pro, the web portal designed by National Highways and
Infrastructure Development Corporation Ltd (NHIDCL) as a common platform to bring cement
buyers and sellers together. It aims to achieve:
o Preventing mismatch of demand and supply.
o Reduction of project execution delays by just in time Supply.
o Mitigating Cost Overruns due to Price Hedging through ceiling price mechanism. Maximum
Increment of 10% permitted YoY basis.
o Avoiding delay in supply by having Online Payment mechanism.e) Promoting Transparency in the
market due to publicly visible Prices.
 The portal has recently been upgraded as INAM-Pro+ to include the A to Z of construction materials,
equipments/machinery and services which would include Purchase/Hiring/Lease of new/used products
and services in following domains:
o Construction Materials viz. Cement, Steel, Bitumen, Stone Aggregates, Concrete, Bricks, Wood,
Sanitary Items, Paint etc.
o Infrastructure Machinery viz.HMP, Paver Finisher, Backhoe Loaders etc.
o Intelligent Transport System Equipment
o Road Furniture
o Haulage Vehicles
o Road Safety and Protection Works

Copyright © by Vision IAS


All rights are reserved. No part of this document may be reproduced, stored in a retrieval system or transmitted
in any form or by any means, electronic, mechanical, photocopying, recording or otherwise, without prior
permission of Vision IAS

31 www.visionias.in ©Vision IAS

Вам также может понравиться